Вы находитесь на странице: 1из 69

01- Regarding intracranial hemorrhage in premature infants, which ONE

of the following is TRUE?

A) 90% of hemorrhages occur by day 1


B) The most common site of hemorrhage is intraventricular
C) 50% of patients with intraventricular hemorrhage require ventriculoperitoneal
shunts
D) Subependymal cyst is a sequela [85 %]
E) Acute parenchymal blood is hypoechoic to the choroid plexus

Answer: D
Approximately 50% of affected infants have onset of hemorrhage on the first
postnatal day, an additional 25% on the second day, and an additional 15% on the third
day (Option A is false). By 72 to 96 hours, 80% to 90% of ICH has occurred. The most
common site of hemorrhage in premature infants is the subependymal germinal matrix
that lies in the floor of the lateral ventricle above the caudate nucleus (Option B is
false). The ventricular dilatation arrests or resolves in 50% to 75% of infants with
posthemorrhagic hydrocephalus.
Progressive ventricular dilatation with increased intracranial pressure develops in the
remaining patients. Fewer than 10% of infants with posthemorrhagic ventricular
dilatation require a shunt (Option C is false). As the subependymal clot retracts, it
undergoes central liquefaction. The cavitary areas can involute completely or evolve into
true cysts (Option D is true). Post-hemorrhagic subependymal cysts are relatively
small, measuring between 3 and 5 mm in diameter. These cysts appear to be of little
clinical significance and most disappear within a year.
Acute parenchymal blood usually is hyperechoic to choroid plexus (Option E is false).
The increased echogenicity associated with hemorrhage, especially when involving the
posterior periventricular area, must be differentiated from the periventricular echoes
seen in normal neonates. In general, the echogenicity of the normal periventricular area
is less than that of the choroid plexus. The normal periventricular echoes also have a
homogeneous, brush-like appearance, whereas hemorrhage is more irregular or
lobulated.

Suggested Readings:
Siegel MJ. Brain. In: Siegel MJ, ed. Pediatric Sonography, 3rd ed. Philadelphia:
Lippincott Williams &
Wilkins, 2001; 41-121.
02- Which ONE of the following features is more commonly associated with
intracranial hemorrhage in premature infants than in term infants?

A) Birth trauma
B) Subdural hemorrhage
C) Cerebellar hemorrhage [50 %]
D) Choroid plexus hemorrhage

Answer: C
Approximately 40% of premature infants at less than 32 weeks of gestation or weighing
less than 1,500 g. The causes of intracranial hemorrhage differ in the two groups. In term
infants, hemorrhage is usually related to birth trauma (Option A is false). Other
predisposing factors include asphyxia, vascular malformations, coagulation defects and
cyanotic heart disease. Bleeding can also occur without these risk factors. In premature
infants, hemorrhage is the result of ischemia.
Subdural hemorrhage is more common in term infants than in premature infants
(Option B is false) and is usually related to birth trauma. Supratentorial subdural
hemorrhage results when there is laceration of the falx or tearing of bridging veins.
Posterior fossa subdural bleeds are a result of tentorial lacerations and occipital
osteodiastasis. Intracerebellar hemorrhage is more frequent in premature infants than in
term infants (Option C is true).
In premature infants, the causes include primary hemorrhage, venous infarction, and
extension of intraventricular or subarachnoid hemorrhage into the cerebellum. In term
infants, the cause of bleeding is thought to be a traumatic delivery which results in
laceration of the cerebellum or rupture of major posterior fossa veins or the occipital
sinuses. Choroid plexus hemorrhage is more common in term infants than in premature
infants (Option D is false). The cause of this hemorrhage is not known precisely.

Suggested Readings:
1. Siegel MJ. Brain. In: Siegel MJ, ed. Pediatric Sonography, 3rd ed. Philadelphia:
Lippincott
Williams & Wilkins, 2001; 41-121.
2. Bellah RD. Intracranial hemorrhage and ischemia in the premature infant. In: Bluth
EI, Arger PH,
Benson CB, Ralls PW, Siegel MJ. Ultrasound: A Practical Approach to Clinical Problems.
Thieme. New York. 2000; 503-520.
03- Regarding periventricular leukomalacia in premature infants, which
ONE of the following is TRUE?

A) The regions of the corona radiata and trigone are the most common sites of
involvement [89%].
B) It is a sequela of intraventricular hemorrhage
C) Sonography reliably distinguishes the hemorrhagic and nonhemorrhagic forms
D) Periventricular cysts are the earliest finding

Answer:
In the premature infant, hypoxic-ischemic injury results in periventricular
leukomalacia (PVL). The frequency of PVL ranges from 7 to 22% in autopsy series. The
watershed area of the premature brain is in the area of the corona radiata and also
adjacent to the trigones of the lateral ventricles and to the foramina of Monro (Option A
is true).
Histopathologically, the earliest finding of PVL is coagulation necrosis.
Phagocytosis of the necrotic area follows within 5 to 7 days, resulting in cavitation. The
cavitary areas can involute, leading to gliosis and loss of white matter volume, or in about
15% of cases, they can evolve into larger cysts. The earliest sonographic finding of PVL,
occurring in the first 2 weeks of life, is increased periventricular hyperechogenicity
(Option D is false). The hyperechogenic zone can disappear within 2 to 3 weeks or it
can evolve into periventricular cysts. Long term sequelae of PVL are ventriculomegaly and
prominent sulci, resulting from brain atrophy. Subependymal and intraventricular
hemorrhages can occur in association with PVL, but they are incidental findings. They are
not causes of PVL (Option B is false).
Parenchymal hemorrhage into areas of PVL has been described in about 25% of cases.
The hemorrhage is thought to be a result of reperfusion of ischemic tissue. The
hemorrhagic and nonhemorrhagic forms of PVL cannot be differentiated by sonography
because the degree of echogenicity is similar in both forms (Options C is false).

Suggested Readings:
1. Siegel MJ. Brain. In: Siegel MJ, ed. Pediatric Sonography, 3rd ed. Philadelphia:
Lippincott
Williams & Wilkins, 2001; 41-121.
2. Vannucci RC. Mechanisms of perinatal hypoxic-ischemic brain damage. Semin
Perinatol 1993;
17:330-337
04- Regarding the Chiari II malformation, which ONE of the following is
true?

A) It is nearly always associated with failure of neural tube closure [77 %]


B) The tentorial attachment is usually normal
C) Supratentorial abnormalities are uncommon
D) The severity of hydrocephalus nearly always improves after repair of the
meningocele

Answer:
The type II malformation is the most common of the Chiari anomalies. It is seen in
neonates and infants and is nearly always associated with a clinically obvious
myelomeningocele (Option A is true). The type II Chiari malformation is characterized
by caudal displacement of the 4th ventricle, cerebellum, and medulla through the
foramen magnum into the upper spinal canal, which results in elongation and thinning
of the medulla, pons and cervical cord. Thus, patients with this malformation have a small
posterior fossa and abnormally low attachment of the tentorium (Option B is false).
Supratentorial anomalies are common (Option C is false). These anomalies include: an
enlarged massa intermedia partially or totally filling the third ventricle,
disproportionately large occipital horns compared with the frontal horns (colpocephaly),
and anterior and inferior angulation of the frontal horns, referred to as a “bat-wing”
appearance. Anomalies of the corpus callosum are also common and consist of dysplasia
or dysgenesis.
Ventricular enlargement is variable prior to surgical repair of the neural tube defect. After
closure of the meningocele, hydrocephalus is present in the majority of patients and not
uncommonly worsens or first becomes apparent at this time (Option D is false). The
explanation for this is thought to be the closure of the spinal defect, which prior to repair,
allowed an outlet for cerebrospinal fluid drainage.

Suggested Readings:
1. Siegel MJ. Brain. In: Siegel MJ, ed. Pediatric Sonography, 3rd ed. Philadelphia:
Lippincott
05- Concerning vein of Galen malformation in neonates, which ONE of the
following is TRUE?

A) Seizure is the most common clinical manifestation


B) Most are fed by multiple arteries rather than a single artery [47 %]
C) A nidus is typical of this malformation
D) It drains into the superior sagittal sinus
E) Spectral tracings show decreased flow velocities in the arterial supply

Answer:
Classically, neonates with the vein of Galen malformation present soon after birth
with high output congestive heart failure due to shunting through a large arteriovenous
fistula. Older infants and children are more likely to present with headaches or seizures
due to intracranial hemorrhage or an enlarging head due to obstructive hydrocephalus
(Option A is false). The vein of Galen malformation is a congenital vascular
malformation resulting from failure of embryonic arteriovenous shunts to be replaced by
capillaries. Two anatomic types of Galenic malformations have been described: choroidal
and mural. The choroidal type is more common and is characterized by multiple arterial
feeders arising in the midbrain. The mural type is characterized by one or a few arterial
feeders (Option B is true).
Venous drainage is into an aneurysmally dilated vein in the area of the vein of Galen. The
draining vein is not the vein of Galen but is the midline prosencephalic vein. The dilated
vein drains into the straight sinus and then into the torcula (Option D is false).
In neonates, this malformation is a direct arteriovenous fistula without an
interconnecting nidus or tangle of abnormal vessels. In older children, there may be a true
arteriovenous malformation which has a nidus (Option C is false).
On coronal sonograms, the Galenic malformation appears as a well-circumscribed
hypoechoic mass in the midline, posterior to the third ventricle. On sagittal views, the
straight sinus and torcula can also be noted.
Spectral tracings typically show elevated flow velocities in the arterial feeders, dampened
pulsatility of the feeders, and arterialization of venous flow (Option E is false).

Suggested Readings:
1. Brunelle F. Arteriovenous malformation of the vein of Galen in children. Pediatr Radiol
1997;
27:501-513
2. Horowitz MB, Jungreis CA, Quisling RG, Pollack I. Vein of galen aneurysms: a review
and
current perspective. AJNR 1994; 15:1486-1496
06- The earliest sonographic finding seen in a normal pregnancy is:

A) The intradecidual sac sign [89 %]


B) The double decidual sac sign
C) The yolk sac
D) The embryo
E) Fluid in endometrial cavity

Answer:
The intradecidual sac sign refers to the gestational sac which implants within the
endometrium. It is visible by 4 to 4.5 weeks’ gestational age (answer a is correct). The
double decidual sac sign is composed of three distinct layers (decidua basalis, capsularis
and parietalis) and is seen when the gestational sac enlarges. It occurs after the
intradecidual sac sign (answer b is incorrect). The yolk sac is seen by 5 to 5.5 weeks’
gestational age, and the embryo is seen by 6 to 6.5 weeks’ gestational age, both later than
the intradecidual sac sign (answers c and d are incorrect). Fluid in the endometrial cavity
is characteristic of pseudosac seen in ectopic pregnancy.

References:

Chiang G, Levine D, Swire M, McNamara A, Mehta T. The intradecidual sign: IS it reliable


for diagnosis of early uterine pregnancy? AJR 2004; 183:725-731.Levine D. Ectopic
Pregnancy.
Radiology 2007;245(2):385-397.Lyons EA, Levi CS. The First Trimester. In Diagnostic
Ultrasound 3rd edition (ed Rumack) 2005;1069-1125.
07- Which of the following is true regarding dichorionic, diamniotic twins?

A) They are always dizygotic


B) They are at risk for twin-twin transfusion syndrome
C) This is the most common twinning presentation [83 %]
D) They separated by a thin membrane
E) The diagnosis is easiest to make in the 2nd trimester

Answer:
Dizygotic twins represent approximately 70% of all twins (answer c is correct).
All dizygotic twins are by definition dichorionic/diamniotic. Additionally, 30% of
monozygotic twins are dichorionic/diamniotic, which occurs if there has been cleavage of
the embryo early in gestation (answer a is incorrect). Since dichorionic, diamniotic
twins do not share a placenta, there is no sharing of circulation and thus no risk of twin-
twin transfusion syndrome (answer b is incorrect). The membrane separating
dichorionic, diamniotic twins is thick, especially in the first trimester (answer d is
incorrect). The diagnosis is easiest to make in the 1st trimester (answer e is incorrect).

References:

Hertzberg BS, Kurtz AB, Choi HY, et al. Significance of membrane thickness in the
sonographic evaluation of twin gestations. AJR 1987; 148:151-153.Levi CS, Lyons EA,
Martel MJ. Sonsgraphy of Multifetal Pregnancy. In Diagnostic Ultrasound 3rd edition (ed
Rumack) 2005;1185-1214.Kennedy AM. Multiple Gestations. In Diagnostic Imaging:
Obstetrics. 1st edition (ed Woodward) 2005; Section 13 p 2-13.
08- Which of the following would be considered abnormal in a first
trimester pregnancy?

A) A gestational sac > 10 mm without a fetal pole


B) A 3-mm embryo without a heartbeat
C) An embryo with a heart rate of 70 bpm [85 %]
D) A 4-mm yolk sac
E) Not seeing an embryo at 5.0 weeks

Answer:
A gestational sac > 18 mm must have a demonstrable fetal pole, but a fetal pole may
not be visible between 10 and 18 mm (answer a is incorrect). A fetal heartbeat is not
seen until the embryo measures 5 mm (answer b is incorrect). Embryonic bradycardia
is defined as a heartbeat < 100 beats per minute; anything less is very concerning and
requires short term follow-up (answer c is correct).
A yolk sac is not considered abnormal until it reaches 6 mm; a 4-mm yolk sac is normal
(answer d is incorrect). The embryo is usually seen at 6-6-5 weeks (answer e is
incorrect).

References:

Levine D. Ectopic Pregnancy. Radiology 2007;245(2):385-397.Lyons EA, Levi CS. The


First Trimester. In Diagnostic Ultrasound 3rd edition (ed Rumack) 2005;1069-
1125.Kennedy AM. Normal Early Pregnancy and Imaging. In Diagnostic Imaging:
Obstetrics. 1st edition (ed Woodward) 2005; Section 1 p 1-6.
09- The amniotic fluid index is defined as:

A) The sum of the deepest pocket of amniotic fluid in four quadrants of the uterus [93
%]
B) The greatest pocket of amniotic fluid that can be measured
C) The average of four pockets of amniotic fluid measured in four quadrants of the
uterus
D) The largest pocket of amniotic fluid multiplied by 4
E) The subjective assessment of amniotic fluid based on prior examinations

Answer: A
The amniotic fluid index as originally defined is the sum of four measurements of
the deepest pocket of amniotic fluid in each quadrant of the uterus. An arbitrary division
of the uterus into quadrants is made at the time of examination and the largest pocket of
fluid is chosen. It is important that no fetal extremities or cord be included in the
measurement. Literature supports the average AFI to range from 8-16 cm. with values of
less than 5 cm. as definite oligohydramnios and more than 20 cm as polyhydramnios.
These determinations are usually compared with the subjective or “eyeball”
determination of the overall amniotic fluid volume and any discrepancy is an indication
for follow-up studies.

References:

Gramellini D, Fieni S, Verrotti C, et al. Ultrasound evaluation of amniotic fluid volume:


methods and clinical accuracy. Acta Biomed 2004;75 Suppl1:40-44. Magann EF,
Sanderson M,
Martin JN, Chauhan S. The amniotic fluid index, single deepest pocket, and two-diameter
pocket in
normal human pregnancy. Am J Obstet Gynecol 2000; 182:1581-1588.
10- Which one of the following combination of measurements are routinely
obtained to determine gestational age?

A) Biparietal diameter, head circumference, estimated fetal weight and femur length
B) Biparietal diameter, head circumference, abdominal circumference and femur
length [92%]
C) Biparietal diameter, head circumference, femur length and abdominal diameter
D) Biparietal diameter, occipito-frontal diameter, estimated fetal weight and femur
length
E) Biparietal diameter, head circumference, estimated fetal weight and thigh
circumference

Answer:
The four measurements routinely performed for the estimation of gestational age are
the biparietal diameter, bead circumference, abdominal circumference, and femur length
(answer b is correct). While the estimated fetal weight is an important part of the
assessment of fetal growth, it is a value derived from the four measurements and is not
an independent measure of gestational age.

References:

AIUM accreditation: http://www.aium.org/publications/clinical/obstetric.pdf


11- The three minimum structures required in imaging the fetal brain are:

A) Cavum septi pellucidi, cisterna magna and ventricular atrium [88 %]


B) Cavum septi pellucidi, cerebellar vermis and ventricular atrium
C) Cavum septi pellucidi, cisterna magna and choroid plexus
D) Corpus callosum, cisterna magna and ventricular atrium
E) Corpus callosum, third ventricle and ventricular atrium

Answer: A
While evaluation of fetal intracranial structures can sometimes be difficult, most
inctracranial abnormalities can be excluded by documenting a normal cavum septi
pellucidi, cisterna magna, and ventricular atrium. A normal cavum septi pellucidi ensures
that the fetal brain has undergone normal cerebral separation and that there is no
holoprosencencephalic spectrum malformation.
Documentation of a normal cisterna magna indicates that there is no Dandy-Walker
malformation or spinal dysraphism. Ventriculomegaly is a non-specific finding, but
commonly accompanies fetal brain malformations. The ACR requires only
documentation of the ventricular atria and cisterna magna, whereas the AIUM also
requires documentation of the cavum septi pellucidi.

References:

AIUM accreditation: http://www.aium.org/publications/clinical/obstetric.pdf


12- In the fetal chest, documentation of the following is required for AIUM
and ACR accreditation:

A) Cardiothoracic ratio
B) Four-chamber view of the heart [98 %]
C) Outflow tracts
D) Ductus arteriosus
E) Color Doppler image of the ventricular septum

Answer:
For both AIUM and ACR accreditation, a four-chamber view of the heart is required
(answer b is correct). The AIUM guideline recommends, but does not require,
obtaining views of the outflow tracts as part of cardiac screening (answer c is
incorrect). Similarly, while evaluation of the cardiothoracic ratio, the ductus arteriosus,
and the ventricular septum are important in fetal echocardiography, they are beyond the
images required for basic standard obstetrical sonographic accreditation (answers a, d,
and e are incorrect).

References:

AIUM accreditation: http://www.aium.org/publications/clinical/obstetric.pdf


13- Documentation of the following normal structures in the fetal abdomen
are required for AIUM and ACR accreditation:

A) Kidneys, stomach, urinary bladder and liver


B) Stomach, spleen, urinary bladder and liver
C) Liver, kidneys, urinary bladder and cord insertion
D) Stomach, kidneys, urinary bladder and cord insertion [93 %]
E) Cord insertion, kidneys, urinary bladder and liver

Answer:
Both the AIUM and ACR require documentation of the stomach, kidneys, urinary
bladder, and umbilical cord insertion site (answer D is correct). In addition, the AIUM
guideline requires documentation of umbilical cord vessel number. Specific evaluation of
the liver and spleen is not part of the standard obstetrical sonographic examination
(answers a, b, c, and e are incorrect).

References:

AIUM accreditation: http://www.aium.org/publications/clinical/obstetric.pdf


14- When performing CT urography, when is it best to administer IV
Furosemide in addition to IV contrast material?

A) When also giving IV saline


B) When there is likely to be, difficulty visualizing the ureters
C) In all patients, as long as there is no history of allergy to sulfa drugs [84 %]
D) In all patients

Answer: C
Furosemide is recommended as a means to improve the opacification and
distension of the collecting systems and ureters. Furosemide can be used to improve the
opacification and distension of the collecting system, and ureters, in all patients, unless
contraindicated. Furosemide is a sulfa derivative and should be withheld in patients with
sulfa allergy. It is not possible to predict which patients’ collecting systems will not be
opacified and distended in advance of the examination. IV saline is a suitable alternative
but does not add significantly to the benefits of furosemide in improving opacification
and distension of the urinary tract.

Reference:
Silverman SG, Akbar SA, Mortele KJ, Tuncali K, Bhagwat JG, Seifter JL. Multidetector-
row CT urography: comparison of furosemide and saline as adjuncts to contrast medium
for depicting the normal urinary collecting system. Radiology 2006; 240:749-755.
15- In which of the following patients is CT urography (including scans
before and after IV contrast material) indicated?

A) 32-year-old man with flank pain


B) 45-year-old woman with a suspicious renal mass
C) 50-year-old with a suspected urinary leak after radical prostatectomy
D) 70-year-old woman with gross hematuria [82 %]

Answer:
DCT urography provides a comprehensive examination of the entire urinary tract
and is, therefore, used in patients with hematuria, particularly those with a risk factor
such as gross hematuria. CT urography could be used in patients with flank pain,
suspicious renal masses, and suspected post-operative complications; however, other
protocols are likely to be as effective and require less scans (and less radiation exposure).
Flank pain in young patients is likely to be due to urolithiasis that can be diagnosed with
an unenhanced CT scan alone. A suspicious renal mass is best evaluated with a renal mass
protocol and does not require detailed images of the urothelium or IV furosemide. A post-
operative complication in which a urinary leak is suspected can be diagnosed with
excretory phase CT scans alone and does not require unenhanced CT or detailed images
of the kidneys.

Reference:

Grossfeld GD, Wolf JS, Litwin MS, et al. Asymptomatic microscopic hematuria in
adults: summary of the AUA best practice policy recommendations. Am Fam Physician
2001; 63:1145-54.
16- Which of the following is true about upper tract transitional cell
carcinoma on CT urography?

A) It is most commonly a polypoid mass


B) It is most commonly noted as collecting system wall thickening [84 %]
C) It can only be detected when larger than 2 cm
D) It cannot be detected at CT urography

Answer: B
Transitional cell carcinoma is most commonly manifest at CT urography as wall
thickening.

Reference:
Caoili EM, Cohan RH, Inampudi P, Ellis JH, Shah RB, Faerber GJ, Montie JM. MDCT
urography of upper tract urothelial neoplasms. AJR 2005; 184:1873-1881.

17- Which of the following is also administered in addition to IV


gadolinium when performing excretory MR urography?

A) IV saline
B) IV furosemide
C) IV saline and furosemide [68 %]
D) Oral water

Answer: C
IV saline and furosemide are used to optimize gadolinium-enhanced excretory MR
urography. IV saline and furosemide are recommended for gadolinium enhanced
excretory MR urography when the upper tracts are not dilated. They help improve upper
tract distention, increase homogeneity of opacification, and minimize artifacts. Saline
alone is not sufficient. Furosemide should be withheld if there are contraindications.

Reference:
Ergen FB, Hussain HK, Carlos, RC, et al. 3D excretory MR urography: improved image
quality with intravenous saline and diuretic administration. J Magn Reson Imaging 2007;
25:783-789.
18- Which of the following is true about using gadolinium-enhanced MR
urography for the detection of upper tract transitional cell carcinoma?

A) It cannot be used for this purpose


B) It should be performed as the first line imaging test instead of CT urography
C) It can be performed instead of CT urography in patients with compromised renal
function
D) It is a potential alternative to CT urography but the value of MR urography for the
detection of small upper tract transitional cell carcinoma is not established [89 %]

Answer: D
A potential alternative to CT urography, but the value of MR urography for the
detection of small upper tract, transitional cell carcinoma is not established. CT
urography has been shown to be excellent for the detection of upper tract transitional cell
carcinoma and is often used as the first line imaging test for this purpose. MR urography
is a potential alternative to CT urography, but its value for the detection of small upper
tract transitional cell carcinoma is not established. Transitional cell carcinoma manifests
as wall thickening or an intraluminal filling defect. Gadolinium contrast agent should be
avoided in patients with compromised renal function due to the risk of Nephrogenic
Systemic Fibrosis (NSF).

Reference:
Nolte-Ernsting CCA, Adam GB, Gunther RW. MR Urography:
examination, techniques and clinical applications. Eur Radiol 2001; 11:355-372.
19- Which of the following is true about T2-weighted MR urography?

A) It provides functional information about the upper tract (pelvocaliceal system)


B) It is used to evaluate dilated upper tracts (pelvocaliceal systems) [87 %]
C) It is used to evaluate non-dilated upper tracts (pelvocaliceal systems)
D) It requires IV gadolinium

Answer: B
T2-weighted MR urography is used to evaluate the upper tract. T2-weighted MR
urography generates static water images of the urinary tract and does not
require gadolinium. It does not yield functional information about the urinary tract. This
technique is used to evaluate the dilated upper tract and can be performed even in
patients with non-excreting kidneys, but is of limited value for the evaluation of the non-
dilated upper tract. T2-weighted MR urography is usually performed using thin-slice and
thick slab T2-weighted single shot fast spin-echo (SSFSE or HASTE) sequences.

Reference:
Nolte-Ernsting CCA, Adam GB, Gunther RW. MR Urography: examination,
techniques and clinical applications. EurRadiol 2001; 11:355-372.

20- All of the following are changes of cirrhosis that may be seen in the liver
on imaging except:

A) Nodular contour of the liver


B) Atrophy of the anterior segment of the right lobe and medial segment of the left
followed by
hypertrophy of the left lateral segment and caudate
C) Periportal space hypertrophy with fat deposition
D) Dilatation and beading of the biliary tree [88 %]

Answer: D
Dilatation and beading of the biliary tree. This may be seen in association with
underlying liver diseases, which may lead to cirrhosis such as primary sclerosing
cholangitis. However, it is not a finding of cirrhotic changes in the liver parenchyma.

Reference:
Dodd GD et al, Spectrum of Imaging Findings of the Liver in End-Stage Cirrhosis:
Part II, Focal Abnormalities. AJR 1999:173(4)1031-1036
21- A 1.5 cm nodule in a cirrhotic liver that demonstrates hyperintensity on
T1-W imaging, isointensity on T2-W imaging, but without increased
enhancement in the arterial phase of imaging relative to the
surrounding liver parenchyma is likely to represent:

A) Regenerative nodule
B) Dysplastic nodule [65 %]
C) Hepatocellular carcinoma
D) Hemangioma

Answer: B
Dysplastic nodule. The lack of arterial enhancement is suggestive of a benign
nodule; either regenerative or dysplastic. The precontrast T1-weighted hyper intensity
may cause difficulty in identifying enhancement in the arterial phase. By subtracting the
precontrast image from the arterial phase image, the presence of enhancement can be
visualized as increased signal intensity on the subtracted image. Both dysplastic and
regenerative nodules have been reported to have variable appearance on T1 –W imaging.
However, the T1-W hyperintensity is more commonly seen in dysplastic nodules than in
regenerative nodules. Hemangioma would be less likely given the T2-W isointensity
rather than hyperintensity.

Reference:
Krinsky GA, Lee VS MR imaging of cirrhotic nodules. Abdom Imaging. 2000 Sep-
Oct;25(5):471-82.
22- In regard to hepatic fibrosis, which of the following is true:

A) Fibrosis tends to be hypointense on T1 and T2 weighted imaging.


B) Fibrosis is usually associated with volume loss and a retraction of the liver capsule
rather than bulging and mass effect. [88 %]
C) Conventional T1-W, T2-W and post contrast MR imaging is highly sensitive in
detection of early stages of fibrosis.
D) Fibrosis tends to have increased enhancement on arterial phase rather than
delayed phases of imaging post IV gadolinium.

Answer: B
Fibrosis is usually associated with volume loss, which can help differentiate focal
confluent fibrosis from HCC. Fibrosis tends to be hypointense on T1-W imaging, but
hyperintense on T2-W imaging. Conventional MRI is not sensitive to early stages of
fibrosis with signal changes usually only being apparent in later stages. Functional MR
techniques such as DWI and PWI are currently being investigated for evaluation of
fibrosis. Fibrosis tends to have increased signal intensity on delayed rather than arterial
phase imaging relative to surrounding liver parenchyma, although occasionally fibrosis
can show arterial enhancement. If arterial enhancement is associated with focal confluent
fibrosis, it can be difficult to differentiate from HCC and biopsy may be required.

Reference:
Dodd GD et al, Spectrum of Imaging Findings of the Liver in End-Stage
Cirrhosis: Part II, Focal Abnormalities. AJR 1999:173(4)1031-1036.

23- In pseudocirrhosis associated with metastatic breast cancer, the liver


may demonstrate the following patterns of liver contour abnormality:

A) Absent
B) Limited retraction of capsule
C) Widespread retraction of the capsule
D) Diffuse nodularity
E) All of the above [88 %]

Answer: E
All of the above-mentioned patterns of contour abnormality can be seen with metastatic
breast cancer with limited areas of capsular retraction being the most common.

Reference:
Qayyum et al, Clinical Imaging 2007;31(1):6-10.
24- Considering current deaths from chronic liver disease, which condition
is most prevalent as the underlying cause of liver disease:

A) Alcoholic liver disease


B) Nonalcoholic fatty liver disease (NAFLD)
C) Hepatitis C [82 %]
D) Hepatitis B
E) Cryptogenic cirrhosis

Answer: C
Hepatitis C accounts for 35%, Alcoholic liver disease 25%, Hepatitis B 20% NAFLD 15%
and 5% other.

Reference:

Sirlin CB the epidemic of cirrhosis and HCC. webcast 2009. ARRS.

25- Which of the following statements is not true?

A) 90-95 % of Hepatocellular carcinoma occurs in patients with liver cirrhosis.


B) Chronic liver disease is one of the top 10 causes of death in the United States with
peak age group shifting to older patients. [63 %]
C) In the future, NAFLD may account for a larger number of patients with chronic
liver disease due to increasing rates of obesity in the USA.
D) Early stage fibrosis in the liver is associated with increased risk of HCC, but less
than that associated with cirrhosis.

Answer: B
Cirrhosis is currently the eighth leading cause of death in the USA; however, the peak
age group is shifting to younger patients.

Reference:

El-Serag HB, Rudolph KL, Hepatocellular carcinoma: epidemiology and molecular


carcinogenesis. Gastroenterology. 2007 Jun;132(7):2557-76.
26- Which of the following statements is false regarding perfusion
weighted imaging (PWI) of the liver:

A) PWI involves the use of high spatial resolution imaging following the
administration of IV contrast. [73 %]
B) The proportion of arterial to portal venous blood supply to the liver increases in
fibrosis and cirrhosis.
C) An advantage of using PWI techniques in detection of fibrosis and cirrhosis is its
noninvasive nature without risks associated with biopsy.
D) Current disadvantages to use of PWI include long processing time on computer
workstation, misregistration due to patient motion during acquisition, limited
accuracy in patients with early fibrosis, and cannot be used in patients with
decreased renal function.

Answer: A
PWI involves the use of high temporal not spatial resolution imaging following the
administration of IV contrast.

Reference:

Taouli B, Perfusion MRI applications in chronic liver disease. Webcast 2009.


ARRS.Annet L, Materne R, Danse E, Jamart J, Horsmans Y, Van Beers BE. Hepatic
flow parameters measured with MR imaging and Doppler US: correlations with
degree of cirrhosis and portal hypertension. Radiology. 2003 Nov;229(2):409-14.
Hagiwara m et al Advanced liver fibrosis: diagnosis with 3D whole-liver perfusion MR
imaging–initial experience. Radiology. 2008 Mar;246(3):926-34.
27- At baseline evaluation, PET/CT is clinically indicated for which one of
the following circumstances:

A) In all patients with esophageal cancer


B) Evaluation of local tumor extension (the T descriptor in TNM staging)
C) Localization of nodal metastases in the gastrohepatic ligament
D) Detection of clinically unsuspected metastases [69 %]

Answer: D
At baseline evaluation, PET/CT is clinically indicated for which one of the following
circumstances:
(a) In all patients with esophageal cancer
Unfortunately, most patients with esophageal cancer present with late stage or metastatic
carcinoma (1), for which PET-CT is unlikely to provide additional useful information.
(b) Evaluation of local tumor extension (the T descriptor in TNM staging)
Assessment of local tumor invasion is not well performed by PET-CT due to its limited
spatial resolution and the variable FDG uptake of early-stage carcinomas (4).
(c) Localization of nodal metastases in the gastrohepatic ligament
Localization of nodal metastases in the gastohepatic ligament is difficult by any technique
and usually requires correlation of CT, EUS and PET-CT images (8).
(d) Detection of clinically unsuspected metastases
PET/CT can detect clinically unsuspected metastases in up to 29% of patients at baseline
evaluation (5).
28- For esophageal cancer following neoadjuvant therapy, FDG PET/CT

A) has no role
B) is inferior to endoscopic ultrasonography for assessment of local tumor response
C) is useful for detection of new interval metastases not present at baseline imaging
[100 %]
D) always indicates residual viable tumor with persistent hypermetabolic primary
tumor activity

Answer: C
For esophageal cancer following neoadjuvant therapy, FDG PET/CT
(a) has no role
FDG PET/CT can detect new interval metastases in up to 8% of patients following
completion of neoadjuvant therapy and prior to planned esophagectomy (15) and is
therefore a useful test (11).
(b) is inferior to endoscopic ultrasonography for assessment of local tumor response A
meta-analysis comparing the diagnostic accuracy of CT, EUS and PET-CT after
neoadjuvant therapy found that PET-CT and EUS were of comparable accuracy in
predicting the presence or absence of residual viable tumor (6).
(c) is useful for detection of new interval metastases not present at baseline imaging
FDG PET/CT can detect new interval metastases in up to 8% of patients following
completion of neoadjuvant therapy and prior to planned esophagectomy (15) and is
therefore a useful test (11).
(d) always indicates residual viable tumor with persistent hypermetabolic primary tumor
activity. A meta-analysis comparing the diagnostic accuracy of CT, EUS and PET-CT after
neoadjuvant therapy found that PET-CT and EUS were of comparable accuracy in
predicting the presence or absence of residual viable tumor (6). However, residual FDG
activity may be seen secondary to esophageal ulceration following radiotherapy or biopsy
(14).
29- According to the recommendations of the Harmonization Project in
Lymphoma, for which type of lymphoma is a pretreatment FDG-PET
scan recommended, but not necessary?

A) MALT lymphoma
B) Cutaneous T-cell lymphoma
C) Diffuse large B-cell lymphoma [77 %]
D) Marginal zone lymphoma
E) Small lymphocytic lymphoma

Answer: C
According to the recommendations of the Harmonization Project in Lymphoma, for
which type of lymphoma is a pretreatment FDG-PET scan recommended, but not
necessary?
(a) MALT lymphoma
Malt lymphoma has variable uptake on FDG-PET/CT, and a baseline scan should be
performed (Cheson BD, Pfistner B, Juweid ME, et. al. Revised response criteria for
malignant lymphoma. 2007. J Clin Oncol 25: 579-86.)
(b) Cutaneous T-cell lymphoma
Cutaneous T-cell lymphoma has variable uptake on FDG-PET/CT, and a baseline scan
should be performed (Cheson BD, Pfistner B, Juweid ME, et. al. Revised response criteria
for malignant lymphoma. 2007. J Clin Oncol 25: 579-86.)
(c) Diffuse large B-cell lymphoma Diffuse Large B-cell lymphoma is typically intensely
hypermetabolic on FDG-PET/CT, and a baseline scan, although preferred, is not
necessary (Cheson BD, Pfistner B, Juweid ME, et. al. Revised response criteria for
malignant lymphoma. 2007. J Clin Oncol 25: 579-86.)
(d) Marginal zone lymphoma
Marginal zone lymphoma has variable uptake on FDG-PET/CT, and a baseline scan
should be performed (Cheson BD, Pfistner B, Juweid ME, et. al. Revised response criteria
for malignant lymphoma. 2007. J Clin Oncol 25: 579-86.)
(e) Small lymphocytic lymphoma Small lymphocytic lymphoma has variable uptake on
FDG-PET/CT, and a baseline scan should be performed (Cheson BD, Pfistner B, Juweid
ME, et. al. Revised response criteria for malignant lymphoma. 2007. J Clin Oncol 25: 579-
86.)
30- In post-therapy FDG-PET imaging, which one of the following is most
concerning for residual tumor?

A) Large residual mass without FDG uptake


B) Diffuse marrow activity
C) Low-grade (blood pool) activity in a large mass
D) Residual splenic mass with uptake similar to normal spleen
E) Persistent focal marrow uptake [92 %]

Answer: E
In post-therapy FDG-PET imaging, which one of the following is most concerning for
residual tumor?
(a) Large residual mass without FDG uptake
Residual masses without FDG uptake are consistent with treated tumor (Juweid ME,
Stoobants S, Hoekstra OS, et. al. Use of positron emission tomography for response
assessment of lymphoma: consensus of the imaging subcommittee of international
harmonization project in lymphoma. 2007. J Clin Oncol 25:571-8.)
(b) Diffuse marrow activity
Diffuse marrow activity in the post-treatment setting should be attributed to marrow
hyperplasia (Juweid ME, Stoobants S, Hoekstra OS, et. al. Use of positron emission
tomography for response assessment of lymphoma: consensus of the imaging
subcommittee of international harmonization project in lymphoma. 2007. J Clin Oncol
25:571-8.)
(c) Low-grade (blood pool) activity in a large mass
Large masses may sometimes demonstrate low-grade radiotracer activity equivalent to
blood pool, and such activity should not be misinterpreted as a sign of residual lymphoma
(Juweid ME, Stoobants S, Hoekstra OS, et. al. Use of positron emission tomography for
response assessment of lymphoma: consensus of the imaging subcommittee of
international harmonization project in lymphoma. 2007. J Clin Oncol 25:571-8.)
(d) Residual splenic mass with uptake similar to normal spleen
A residual splenic mass equivalent to background spleen in intensity is consistent with
treated tumor (Juweid ME, Stoobants S, Hoekstra OS, et. al. Use of positron emission
tomography for response assessment of lymphoma: consensus of the imaging
subcommittee of international harmonization project in lymphoma. 2007. J Clin Oncol
25:571-8.)
(e) Persistent focal marrow uptake Focal marrow uptake in the post-treatment setting
should be considered suspicious for residual lymphoma (Juweid ME, Stoobants S,
Hoekstra OS, et. al. Use of positron emission tomography for response assessment of
lymphoma: consensus of the imaging subcommittee of international harmonization
project in lymphoma. 2007. J Clin Oncol 25:571-8.)
31- In transformed lymphoma, which one of the following statements is
true?

A) Chronic Lymphocytic Leukemia (CLL) commonly converts to high-grade


lymphoma
B) In Richter Transformation, the pathology is usually Mantle Cell Lymphoma
C) The largest nodal masses in transformed CLL are always involved with high-grade
lymphoma
D) PET can differentiate between sites of low and high-grade tumor in transformed
CLL [77%]
E) Patients with Richter Transformation have an excellent prognosis

Answer: D
In transformed lymphoma, which one of the following statements is true?
(a) Chronic Lymphocytic
Leukemia (CLL) commonly converts to high-grade lymphoma Transformation is an
uncommon occurrence, with a rate of approximately 4% (Bruzzi JF, Macapinlac H,
Tsimberidou AM, et. al. Detection of Richter’s transformation of chronic lymphocytic
leukemia by PET/CT. 2006. J Nucl Med 47:1267-73.)
(b) In Richter Transformation, the pathology is usually Mantle Cell Lymphoma
When transformation occurs, the histopathology is typically diffuse large B-cell
lymphoma (Bruzzi JF, Macapinlac H, Tsimberidou AM, et. al. Detection of Richter’s
transformation of chronic lymphocytic leukemia by PET/CT. 2006. J Nucl Med 47:1267
73.)
(c) The largest nodal masses in transformed CLL are always involved with high-grade
lymphoma Transformation can occur in any size node; one of the strengths of FDG-
PET/CT in patients with suspected transformation is its ability to differentiate nodes
based on metabolic activity rather than size (Bruzzi JF, Macapinlac H, Tsimberidou AM,
et. al. Detection of Richter’s transformation of chronic lymphocytic leukemia by PET/CT.
2006. J Nucl Med 47:1267-73.)
(d) PET can differentiate
between sites of low and high-grade tumor in transformed CLL
Areas of high-grade tumor show more intense tracer uptake on FDG-PET/CT than areas
of indolent disease, and PET has been shown to differentiate these two processes (Bruzzi
JF, Macapinlac H, Tsimberidou AM, et. al. Detection of Richter’s transformation of
chronic lymphocytic leukemia by PET/CT. 2006. J Nucl Med 47:1267-73.)
(e) Patients with Richter Transformation have an excellent
Prognosis The prognosis of Richter Transformation is poor (Bruzzi JF, Macapinlac H,
Tsimberidou AM, et. al. Detection of Richter’s transformation of chronic lymphocytic
leukemia by PET/CT. 2006. J Nucl Med 47:1267-73.)
32- In lung cancer staging, which one of the following statements is true
regarding PET/CT?

A) It is more useful than CT in determining the extent of the primary tumor (T stage).
B) It improves detection of distant metastasis compared to conventional imaging (i.e.,
CT, MRI). [85 %]
C) It has reduced futile (unnecessary) lung cancer surgeries by 5%.
D) All lung cancers greater than 2 cm exhibit FDG activity.
E) Its lymph node specificity for malignancy is 59-89%.

Answer: B
In lung cancer staging, which one of the following statements is true regarding PET/CT?
(a) It is more useful than CT in determining the extent of the primary tumor (T stage).
It has limited value because the tumor margins are not clearly defined by the FDG activity
secondary to blooming artifacts, and therefore the extent and invasion by the tumor (T)
is not as good as CT.
(b) It improves detection of distant metastasis compared to conventional imaging (i.e.,
CT, MRI). Correct Answer
(c) It has reduced futile (unnecessary) lung cancer surgeries by 5%. Futile surgery is
reported to be reduced by 21 – 41%.
(d) All lung cancers greater than 2 cm exhibit FDG activity.
FDG can be negative in some tumors, such as bronchioloalveolar cell carcinoma,
regardless of size.
(e)
Its lymph node specificity for malignancy is 59-89%.
Lymph node specificity is reported to be 94 – 100%. CT alone is 59 – 89%.

Reference:

Van Tinteren H, Hoekstra OS, Smit EF, et al. Effectiveness of positron emission
tomography in the preoperative assessment of patients with suspected non-small-cell
lung cancer: the PLUS multicenter randomized trial. Lancet 2002;359(9315):1388-93.
33- In regards to re-assessing lung cancer patients, which one of the
following is true?

A) PET/CT cannot differentiate radiation fibrosis from recurrent tumor within the
radiation field.
B) Conventional CT is not useful in assessing tumor response using RECIST criteria.
C) PET/CT has a low utility in detecting second primary tumors or distant metastasis.
D) PET/CT has no value in assessing tumor response to therapy.
E) 4D PET/CT is a method to correct for motion artifacts in calculating SUV in
PET/CT imaging. [77 %]

Answer: E
In regards to re-assessing lung cancer patients, which one of the following is
true?
(a) PET/CT cannot differentiate radiation fibrosis from recurrent tumor within the
radiation field. FDG activity is high in recurrent tumor and there is no or very little activity
in fibrosis.
(b)Conventional CT is not useful in assessing tumor response using RECIST criteria
RECIST criteria are based on conventional CT imaging
(c) PET/CT has a low utility in detecting
second primary tumors or distant metastasis. Second primary tumors are detected in
approximately 5% of lung cancer patients
(d) PET/CT has no value in assessing tumor response to therapy. PET/CT is proving to
be very useful in assessing tumor response and new guidelines are under development for
RECIST applications.
(e) 4D PET/CT is a method to correct for motion artifacts in calculating SUV in PET/CT
imaging. Correct Answer

Reference:

Munden RF, Swisher SG, Stevens CW, Stewart DJ. Imaging of the patient with non-
small cell lung cancer. Radiology 237:803-818, 2005
34- Which is a potential cause of acetabular labral tear?

A) Trauma
B) Femoral-acetabular impingement
C) Hip dysplasia
D) All of the above [94 %]

Answer: D
Tears of the acetabular labrum can result from a variety of causes. They can result
from acute trauma such as a hip subluxation injury, or repetitive trauma as occurs in
femoral-acetabular impingement. Degenerative tears can occur from excess stress on the
labrum as seen in patients with a shallow acetabulum from development dysplasia of the
hip.

References:

1. Petersilge CA, Haque MA, Petersilge WJ, Lewin JS, Lieberman JM, Buly R.
Acetabular labral tears: evaluation with MR arthrography. Radiology. 1996
Jul;200(1):231-235.
2. Kassarjian A, Belzile E. Femoroacetabular impingement: presentation, diagnosis,
and management. Semin Musculoskelet Radiol. 2008 Jun;12(2):136-145.
35- A possible sign of femoral-acetabular impingement includes:

A) An osseous prominance (‘bump’) at the femoral head-neck junction [93 %]


B) Upturn of the lateral acetabular roof
C) A shallow acetabular socket
D) Acetabular anteversion

Answer: A
One of the two main types of femoral-acetabular impingement (FAI) is cam FAI,
which results from an osseous bump at the head-neck junction. An upturned lateral
acetabular roof would allow more free range of motion of the femoral head and make FAI
less likely, as would a shallow acetabular socket. Acetabular retroversion is associated
with the 2nd type of FAI, pincer-type. Anteversion however is not associated with FAI.

Reference:

Tannast M, Siebenrock KA, Anderson SE. Femoroacetabular impingement:


radiographic diagnosis–what the radiologist should know. AJR Am J Roentgenol. 2007
Jun;188(6):1540-52.

36- The best imaging exam for acetabular labral tear is:

A) Ultrasound
B) MRI
C) CT
D) MR arthrogram [89 %]

Answer: D
MR arthrography is the most sensitive and accurate imaging exam for identifying labral
tears, with an accuracy of 90-95%. Conventional MR is much less accurate as is CT.
Ultrasound has not been proven to be better than MR arthrography for labral tears.

Reference:

Toomayan GA, Holman WR, Major NM, Kozlowicz SM, Vail TP. Sensitivity of MR
arthrography in the evaluation of acetabular labral tears. AJR Am J Roentgenol 2006;
186:449–453.
37- Which of the following is true in regard to MR arthrography of the hip?

A) Joint distention is achieved by injection of more than 20 ml of a dilute solution of


gadolinium.
B) Posterior acetabular labral tears are the most commonly encountered labral tear.
C) MR arthrography has a high sensitivity and accuracy of at least 90% for labral
tears. [93%]
D) Labral tears are recognized by interposition of contrast material between the
acetabulum and labrum in the posteroinferior quadrant of the acetabulum.
E) In asymptomatic individuals, the labrum is typically rounded or blunted.

Answer: C
a. Adequate hip joint distension for MR arthrography can be achieved with a
volume of 12-15 cc. False
b. In several studies, around 90% of acetabular labral tears occur in the anterior superior
labrum. False
c. A study by Czerny et al. reported a sensitivity of 90% and a specificity of 91% for MR
arthrography of labral tears. True
d. Labral tears appear on MR arthrography as either increased signal that extends to the
surface of the labrum, or abnormal interposition of contrast between the labrum and
acetabulum. Interposition of contrast material at the acetabular labral junction can also
be seen normally posteroinferiorly where it represents a normal recess. Some authors
believe that a shallow smooth sulcus can occur at other regions involving the acetabular
labral junction, including the anterior superior acetabulum. False
e. In asymptomatic individuals the most common appearance of the labrum is triangular,
seen in 66%-80% of people at MR arthrography. A minority of asymptomatic individuals
may have a blunted or rounded shape to the labrum, and this is more common as people
get older.

References:

1. Byrd JW, Jones KS. Diagnostic accuracy of clinical assessment, magnetic resonance
imaging, magnetic resonance arthrography, and intra-articular injection in hip
arthroscopy patients. Am J Sports Med. 2004 Oct-Nov;32(7):1668-74.
38- The most common cause of an internal snapping hip is due to:

A) Labral tear
B) Intraarticular loose body
C) Iliopsoas tendon [79 %]
D) Iliofemoral ligament
E) Iliotibial band

Answer: C
a. internal and intra-articular. Labral tears and loose bodies are categorized as the
intraarticular type when they cause a painful snapping hip. False
b. Loose bodies are categorized as the intraarticular type when they cause a painful
snapping hip. False
c. The main cause of the internal type of the snapping hip syndrome is due to the iliopsoas
tendon passing over the iliopectineal eminence at the anterior acetabular rim. True
d. The iliofemoral ligaments are an uncommon cause of internal snapping hip syndrome.
e. The iliotibial band snapping as it passes over the greater trochanter is considered an
external type of snapping hip syndrome. False

References:
1. Schaberg JE, Harper MC, Allen WC. The snapping hip syndrome. Am J Sports Med
1984; 12:361-365.
2. Cardinal E, Buckwalter KA, Capello WN, Duval N. US of the snapping iliopsoas tendon.
Radiology. 1996 Feb;198(2):521-2.
3. Pelsser V, Cardinal E, Hobden R, Aubin B, Lafortune M. Extraarticular snapping hip:
sonographic findings. AJR Am J Roentgenol. 2001 Jan;176(1):67-73.
39- By MRI, osteitis pubis manifests as:

A) Unilateral or grossly asymmetric pubic ramus bone marrow edema


B) A ‘secondary cleft’ sign on T2 weighted coronal images
C) Subchondral marrow edema spanning the anterior – posterior symphysis with or
without osseous productive change [89 %]
D) A joint effusion extending from the pubic symphysis into the inguinal canal

Answer: C
Osteitis pubis appears as marrow edema of the pubic bones on both sides of the
symphysis. Osteitis pubis shows MRI characteristics similar to osteoarthritis, with
reciprocal bone marrow edema in a subchondral location. A secondary cleft sign indicates
a disruption of the rectus abdominis/abductor aponeurosis or an abductor tendon
avulsion, and is a more severe injury. Joint effusion extending into the inguinal canal is
not seen with osteitis pubis.

Reference:
Athletic pubalgia and the “sports hernia”: MR imaging findings. Zoga AC, Kavanagh EC,
Omar IM, Morrison WB, Koulouris G, Lopez H, Chaabra A, Domesek J, Meyers WC.
Radiology. 2008 Jun;247(3):797-807.
40- Optimization of MRI in the setting of clinical athletic pubalgia
includes?

A) The use of dynamic imaging with and without Valsalva


B) Pre-and post-intravenous contrast T1 weighted fast field echo/gradient echo
acquisitions
C) Positioning the patient prone for reduction of respiratory motion artifact
D) A dedicated surface receiver coil positioned over the pubic symphysis [90 %]

Answer: D

The optimal protocol includes both high resolution, small field of view sequences with the
surface coil turned on for symphysis pathology as well as large field of view sequences of
the pelvis using the built-in body coil for remote, confounding lesions. Dynamic imaging
is used to image inguinal hernias, which are different than ‘sports hernias.’ Intravenous
contrast and prone positioning are not necessary to image athletic pubalgia.

Reference:
Athletic pubalgia and “sports hernia”: optimal MR imaging technique and findings. Omar
IM, Zoga AC, Kavanagh EC, Koulouris G, Bergin D, Gopez AG, Morrison WB, Meyers WC.
Radiographics. 2008 Sep-Oct;28(5):1415-38.
41- The lesion most likely to be identified clinically as a “sports hernia” is
a(n)?

A) Rectus abdominis / adductor aponeurosis tear or detachment. [96 %]


B) Tear of the posterior wall of the inguinal canal
C) Breech of the external oblique aponeurosis
D) Adductor longus tendon avulsion

Answer: A
Rectus abdominis / adductor aponeurosis tear or detachment is the usual etiology
of athletic pubalgia. The lateral edge of the caudal rectus abdominis tendon is positioned
just deep to the posterior wall of the superficial inguinal ring. Tear of the posterior wall of
the inguinal canal or tears of the external oblique aponeurosis are uncommon causes of
groin pain in athletes. Adductor longus tendon avulsion can be seen in athletes with groin
pain, but may present more with more distal symptoms.

Reference:
Experience with “sports hernia” spanning two decades. Meyers WC, McKechnie A,
Philippon MJ, Horner MA, Zoga AC, Devon ON. Ann Surg. 2008 Oct;248(4):656-65.

42- A rectus abdominis / adductor aponeurotic plate disruption will


manifest on MRI as?

A) Unilateral detachment of the rectus abdominis / adductor aponeurosis causing


enlargement of the superficial inguinal ring
B) A confluent detachment of bilateral rectus abdominis tendons from their caudal
pubic insertions [48 %]
C) Displacement of the interpubic disk at the pubic symphysis
D) Fatty atrophy of the rectus abdominis muscle, unilateral or bilateral

Answer: B
This bilateral lesion can lead to instability at the pubic symphysis causing rapid
evolution of osteitis pubis, and is unlikely to respond long term to conservative therapies.
Unilateral detachment is less common. Displacement of the pubic symphysis disc is
uncommon, while fatty atrophy of the rectus abdominis in athletes is generally related to
a unilateral aponeurosis lesion.

Reference:
Anatomy, pathology, and MRI findings in the sports hernia. Shortt CP, Zoga AC,
Kavanagh EC, Meyers WC. Semin Musculoskelet Radiol. 2008 Mar;12(1):54-61.
43- The most frequently encountered confounding lesion for pubic
symphysis osseous or tendinous injury in the setting of groin pain is?

A) Inguinal hernia
B) Obturator internus or iliopsoas bursitis
C) Internal derangement of the hip/labral tear [76 %]
D) Adnexal lesions

Answer: C
There are many confounders about the pelvis that may manifest as activity-induced groin
pain, but labral tear or hip chondral lesions are the most common.

Reference:
Spectrum of MRI findings in clinical athletic pubalgia. Zajick DC, Zoga AC, Omar IM,
Meyers WC.
Semin Musculoskelet Radiol. 2008 Mar;12(1):3-12.

44- In a postmenopausal woman with abnormal vaginal bleeding who is


not receiving hormone replacement therapy, which of the following
endometrial thickness cutoff criteria is used to optimize accuracy for
detecting cancer?

A) ≥ 4 mm.
B) ≥ 5 mm. [96 %]
C) ≥ 6 mm.
D) ≥ 7 mm.
E) ≥ 8 mm.

Answer: B
Endovaginal sonography is the first-line screening technique for endometrial
cancer in the workup of postmenopausal women receiving hormone replacement therapy.
A thickness of 8 mm is considered the upper limit of normal if the patient is
asymptomatic. However, if the patient reports postmenopausal bleeding, a thickness
cutoff of ≥ 5 mm is used. Option B is the best response.
Meta-analysis of the diagnostic performance of endovaginal sonography in detecting
endometrial cancer [1] has shown that a cutoff criterion of ≥ 5 mm results in 96%
sensitivity and 77% specificity and shows optimum overall accuracy. Higher thickness
cutoff criteria result in improved specificity but significant decreases in sensitivity,
whereas lower-thickness cutoff criteria do not significantly increase sensitivity. Options
A, C, D, and E are not the best responses.
45- All of the following increase a woman’s risk for endometrial
hyperplasia and cancer EXCEPT which one?

A) Multiparity. [96 %]
B) Obesity.
C) Diabetes.
D) Hypertension.
E) Tamoxifen exposure.

Answer: A
Endometrial hyperplasia and cancer are caused by unopposed estrogen stimulation. Risk
factors include endogenous or exogenous exposure to estrogen, tamoxifen use,
nulliparity, obesity, hypertension, and diabetes [2]. Options B, C, D, and E are not the
best responses. Multiparity is not a risk factor for endometrial hyperplasia and cancer.
Option A is the best response.

46- Which of the following statements regarding women receiving


tamoxifen is FALSE?

A) Tamoxifen causes an increase in the prevalence of endometrial polyps,


hyperplasia, and carcinoma.
B) Postmenopausal women taking tamoxifen usually show endometria that are
thicker than in control subjects.
C) Endovaginal sonography is an accurate tool for diagnosing endometrial
abnormalities in this patient population. [95 %]
D) Subendometrial cystic changes can often simulate endometrial thickening on
transvaginal sonography.
E) What should be considered normal endometrial thickness in asymptomatic
women on tamoxifen is controversial.

Answer: C
Tamoxifen has a weak estrogenic effect in the uterus and causes an increased
prevalence of endometrial polyps, hyperplasia, and carcinoma. Option A is not the best
response. Subendometrial cysts, a finding associated with tamoxifen exposure, can often
simulate endometrial thickening on transvaginal sonography, decreasing its diagnostic
accuracy. Option D is not the best response.
Furthermore, several disorders, such as polyps and carcinoma, may coexist, limiting
the usefulness of endovaginal sonography in the diagnosis of specific abnormality [3].
Option C, which is not true, is the best response. Sonohysterography may help
differentiate subendometrial from endometrial abnormalities and guide proper
diagnostic workup. Postmenopausal women taking tamoxifen often have endometria that
are thicker than those of control subjects [4], and most of them are asymptomatic. Option
B is not the best response. What constitutes the normal endometrial thickness for this
group of patients is controversial, with 5–8 mm having been proposed [5–7]. Option E is
not the best response.
47- The differential diagnosis of focal endometrial abnormality seen on
sonohysterography includes which of the following?

A) Polyp.
B) Hyperplasia.
C) Carcinoma.
D) Subendometrial fibroid.
E) All of the above. [97 %]

Answer: E
Polyps are a common cause of focal endometrial thickening. On sonohysterography,
polyps appear as echogenic, smooth, intracavitary masses outlined by fluid. The point of
attachment does not disrupt the endometrial lining. Cystic spaces corresponding to
dilated glands filled with proteinaceous fluid, or heterogeneous echotexture as a result of
hemorrhage, infarction, or inflammation, can be seen.
Although diffuse thickening is the most common appearance of endometrial hyperplasia
and cancer, both can be focal on occasion. Subendometrial fibroids can usually but not
always be distinguished from endometrial disorders by the appearance on
sonohysterography. Subendometrial fibroids are typically hypoechoic, well-defined solid
masses that show acoustic attenuation. Most important, they show an overlying layer of
echogenic endometrium [8]. Options A–D are all correct, so option E is the best
response.
48- In differentiating focal endometrial disorders (e.g., polyp) from a
subendometrial disorder (e.g., fibroid) on sonohysterography, which
of the following statements is FALSE?

A) Polyps are frequently multifocal, whereas fibroids are usually solitary. [94 %]
B) Polyps usually show a narrow base, whereas fibroids have a broad base of
attachment to the uterine wall.
C) Polyps are typically echogenic like normal endometrium, whereas fibroids are
typically hypoechoic like normal myometrium.
D) The normal endometrial lining underlies the base of a polyp, whereas it overlies
the surface of a fibroid.
E) On color Doppler imaging, polyps show a single feeding vessel, whereas fibroids
show a diffuse network of vessels.

Answer: A
On sonohysterography, endometrial polyps are typically echogenic, like normal
endometrium, and show a narrow attachment to the normal endometrial lining at its base
[9]. Option B is not the best response. A single feeding vessel is sometimes seen on color
Doppler sonography. In contrast, sub endometrial fibroids are typically hypoechoic, like
normal myometrium, and show a broad base of attachment to the myometrial wall with
the normal endometrial lining overlying its surface [8]. Option C is not the best response.
Fibroids show a hyper vascular network of vessels on color Doppler sonography [10].
Option E is not the best response. The key to differentiating the two entities is ascertaining
the location of the endometrial lining with regard to the lesion. The normal endometrial
lining underlies the base of a polyp, whereas it overlies the surface of a fibroid. Option D
is not the best response. Whether a focal lesion is solitary or multiple on
sonohysterography does not distinguish between an endometrial or sub endometrial
process. Option A, which is not true, is the best response.
49- An endovaginal sonography examination depicts the endometrium of
a postmenopausal woman with vaginal bleeding, abnormally thick,
measuring 2.1 cm, and contains multiple cysts. There is a history of
several years of tamoxifen exposure. Which of the following is the
LEAST LIKELY diagnosis?
A) Polyp.
B) Hyperplasia.
C) Carcinoma.
D) Subendometrial fibroid. [93 %]
E) Subendometrial cysts.

Answer: D
The endometrium is abnormally thick, measuring 2.1 cm, and contains multiple
cysts. Chronic exposure to tamoxifen results in an increased likelihood of endometrial
disorders such as polyps, hyperplasia, and carcinoma [4]. Options A, B, and C are not the
best responses. These disorders areoften detected as abnormal endometrial thickening,
sometimes with cystic changes. In addition, patients taking tamoxifen can show
subendometrial cysts that mimic endometrial thickening on transvaginal sonography
[11]. Option E is not the best response. The appearance of this endometrium is least
consistent with an underlying submucosal fibroid that typically appears hypoechoic,
homogeneously solid, and well circumscribed. Option D is the best response.

50- A sonohysterography examination depicts a a hypoechoic, well-


defined solid mass focal lesion. Which of the following is the MOST
LIKELY diagnosis?
A) Polyp.
B) Hyperplasia.
C) Carcinoma.
D) Subendometrial fibroid. [97 %]

Answer: D
The lesion is a hypoechoic, well-defined solid mass with a broad base of attachment.
Most important, it shows an overlying layer of echogenic endometrium, indicating that
this lesion is subendometrial in location. This appearance is most consistent with a
subendometrial fibroid. Option D is the best response. Polyp, hyperplasia, and
carcinoma are endometrial disorders. Options A, B, and C are not the best responses.
51- Which of the following statements regarding diagnostic tools for
endometrial disorders is TRUE?
A) Nonfocal biopsy to detect cancer should be performed after a negative workup for
a focal abnormality.
B) Sonohysterography is more accurate than hysteroscopy for detecting focal
endometrial disorders.
C) Endovaginal sonography is the most sensitive test for endometrial cancer detection
in postmenopausal women. [84 %]
D) Endovaginal sonography is highly sensitive in detecting endometrial disorders in
the premenopausal woman.
E) MRI is replacing sonohysterography as a diagnostic tool for endometrial disorders.

Answer: C
Because endometrial cancer and hyperplasia, which are non-focal abnormalities,
constitute the only potentially life-threatening endometrial disorders, endovaginal
sonography, followed by non-focal endometrial biopsy when indicated, should be
performed early in the diagnostic workup of abnormal bleeding to evaluate for these
disorders. Only after these prove negative should focal, more likely benign causes be
sought to explain the bleeding. Option A is not the best response.
Sonohysterography and hysteroscopy are two methods for detecting focal lesions
and are similar in sensitivity and specificity [12, 13]. Option B is not the best response.
Endovaginal sonography is highly effective in screening for cancer in the postmenopausal
population using a thickness cutoff of ≥ 5 mm. Option C is the best response. In the
premenopausal population, endovaginal sonography is a useful tool for identifying mural
abnormalities such as fibroids and adenomyosis. However, its performance in detecting
endometrial disorders is suboptimal, with a sensitivity of 67% and a specificity of 75%
[14]. Option D is not the best response. The role of MRI in the workup of abnormal vaginal
bleeding is limited. Option E is not the best response.
52- Sonohysterography can be appropriately used in evaluating women
with abnormal bleeding for all of the following purposes EXCEPT
which one?

A) Evaluate endometrium not visualized or poorly visualized on endovaginal


sonography.
B) Evaluate women with abnormal vaginal bleeding and normal findings on
endovaginal sonography for underlying endometrial disorders.
C) Distinguish abnormality seen on endovaginal sonography as endometrial versus
subendometrial.
D) Determine the size and location of focal lesions to plan hysteroscopic resection.
E) Characterize an endometrial lesion as benign or malignant. [97 %]

Answer: E

Sonohysterography distinguishes endometrial from subendometrial abnormalities.


Options C is not the best response. Once a lesion has been characterized as endometrial,
imaging cannot reliably exclude malignancy. Focal endometrial hyperplasia or carcinoma
can mimic a sessile polyp, and foci of atypical hyperplasia are sometimes found in polyps.
Thus, endometrial lesions should undergo histologic evaluation. Option E, which is
not true, is the best response. Sonohysterography is useful for detecting, localizing,
and sizing focal endometrial disorders before hysteroscopic resection. Option D is not the
best response. It is more sensitive than endovaginal sonography for detecting focal
endometrial disorders, especially in premenopausal women or those who have undergone
long-term tamoxifen therapy [15]. Option B is not the best response.
Sonohysterography can be used to evaluate the endometrium when endovaginal
sonography fails to adequately visualize it, often because of distortion by fibroids. Option
A is not the best response.
53- When a bone tumor is suspected, what role does conventional
radiography play in the workup?

A) Conventional radiography is the first choice for initial imaging. [100 %]


B) Conventional radiography is limited because of poor lesion characterization.
C) Conventional radiography is limited because of poor lesion detection.
D) Conventional radiography is rarely useful.

Answer: A
Conventional radiographs are the mainstay for initial evaluation of bone lesions [1,
2]. Radiographs provide important information regarding the underlying pathology of
bone tumors. Particularly, the location in the skeleton, location in the bone, rate of tumor
growth (aggressiveness), and lesion characterization, including mineralization, are key
factors leading to a diagnostic conclusion. Option A is the best response.
Conventional radiographs are very useful to characterize lesions, especially with regard
to aggressiveness of the lesion and associated lesion matrix. Option B is not the best
response. Conventional radiographs are useful for lesion detection. Metastatic bone
surveys are commonly used in clinical practice. Option C is not the best response.
Conventional radiographs are commonly used when working up suspected bone tumors.
Their specificity is excellent because of their ability to characterize lesion location, lesion
rate of growth, and associated tissue, which significantly narrows the differential
diagnosis in many cases. Option D is not the best response.

54- Advantages of CT over MRI for bone tumor characterization include


evaluation of all of the following features EXCEPT:

A) Cortical integrity.
B) Neurovascular invasion. [92 %]
C) Matrix mineralization.
D) Periosteal reaction.

Answer: B
CT for characterization of tumors is superior to MRI when assessing certain tumor
characteristics. However, MRI is better than CT for evaluation of the soft tissues [1]. This
includes the presence or absence of neurovascular invasion [2]. Option B, which is
false, is the best response. CT is superior to MRI for assessing bone and other
mineralized structures [2] and for assessing cortical integrity, matrix mineralization, and
periosteal reaction. Options A, C, and D are correct and are not the best responses.
55- Limb-sparing surgical techniques are used for all of the following
reasons EXCEPT:

A) Improved or equal long-term survival rates compared with conventional surgical


techniques.
B) Improved functional results.
C) Proven efficacy of metastasectomy.
D) Ease of surgical procedure. [92 %]

Answer: E
Limb-sparing surgical techniques are used for many reasons. These surgical
techniques are complicated and were designed for optimum patient outcome, not for the
ease of the surgical procedure [1]. Option D, which is false, is the best response.
Outcomes for limb-sparing surgical techniques show improved or equal long-term
survival rates when compared with conventional surgical techniques. Option A, which is
true, is not the best response. Limb-sparing surgical techniques are most valuable because
of the improved functional results for patients [3]. Option B, which is true, is not the best
response. Limb-sparing surgical techniques are also valuable because surgical resection
of distant metastases has been proven to be effective. Option C, which is true, is not the
best response.

56- For the imaging of bone tumors, MRI has the advantage over CT for all
of the following reasons EXCEPT:

A) Superior soft-tissue contrast resolution.


B) Improved assessment of skip lesions.
C) Need to use IV contrast material. [92 %]
D) Lack of exposure to ionizing radiation.

Answer: C
MRI has some advantages over CT when evaluating bone tumors. However, the need
to use IV contrast material is not one of these advantages [1]. Most bone lesions can be
accurately characterized on CT without the use of IV contrast material. Option C, which
is false, is the best response. MRI is superior to CT with respect to soft-tissue contrast
resolution [2]. Option A, which is true, is not the best response. MRI also well evaluates
the marrow space within the bone, providing a more sensitive method for the detection
of skip metastases. This is also an advantage. Option B, which is true, is not the best
response. The lack of exposure to ionizing radiation in MRI is also an advantage,
especially when evaluating children. Option D, which is true, is not the best response.
57- In a patient with an intramedullary bone lesion, a primary sarcoma
can be excluded in the presence of:

A) Homogeneous density of –20 HU on CT. [100 %]


B) History of previously resected breast cancer.
C) Similar signal on in-phase and opposed-phase MR images.
D) Patient age > 65 years.

Answer: A
If a bone “lesion” has internal fat density and a uniform appearance, it is likely a focal
area of trabecular scarcity related to osteoporosis or an intraosseous lipoma [6]. Tumors
generally replace the marrow fat from the intramedullary space rather than filter
throughout the marrow. A liposarcoma may have some internal fat but would not appear
homogeneous or be within a bone unless it is metastatic. Option A is the best
response. Even if a patient has a history of a primary malignancy, one cannot assume
that a new bone lesion is a metastasis without a biopsy. If a treatment regimen is begun
for the wrong type of tumor, the disease may progress to a more advanced stage before
the correct diagnosis is made and the correct treatment is begun. Option B is not the best
response.
Similar low signal on in phase and opposed-phase gradient-echo MR images is a sign
that a water containing lesion has replaced all marrow fat, which can be seen with both
malignant and benign tumors. When a lesion loses signal on the opposed-phase image
compared with the in-phase image, it is most likely benign, representing either
hematopoietic marrow or marrow edema [7, 8]. Option C is not the best response.
Although it is true that a bone or soft-tissue tumor in a patient older than 65 years is more
likely to be a metastasis or myeloma, many primary sarcomas of bone and soft tissues
occur in elderly patients. Option D is not the best response.
58- Reasons for a non-diagnostic or false-negative bone biopsy include
sampling all of the following EXCEPT:

A) Areas of adjacent reactive periosteal new bone formation.


B) Foci of tumor necrosis shown on enhanced imaging.
C) Areas with tumor vascularity seen on Doppler ultrasound. [92 %]
D) Areas of hemorrhage in or adjacent to the tumor.

Answer: C
The finding of blood-flow Doppler signal in a portion of a mass is useful for identifying
a target for biopsy that contains viable tissue [6]. Option C, which is false, is the best
response. In sclerotic bone lesions, some of the density is due to reactive bone formation
and does not contain tumor cells. A needle biopsy of a reactive portion of a lesion can have
a non-diagnostic result. Diagnostic yield can be increased with larger needles and more
biopsy passes [9]. Option A, which is true, is not the best response.
If a biopsy of a necrotic lesion is attempted, the solid and viable portions should be
sought as a target. Necrotic areas may not contain enough tumor cells for a successful
diagnosis [10]. Option B, which is true, is not the best response. Regions of masses that
appear hemorrhagic on MRI or CT often have less diagnostic yield than solid viable areas
of abnormal tissue. Option D, which is true, is not the best response.

59- Which of the following statements is TRUE regarding tumor


cryoablation versus radiofrequency ablation?

A) Cryoablation is more painful than radiofrequency ablation.


B) Cryoablation is easier to monitor than radiofrequency ablation. [96 %]
C) Cryoablation is more harmful to fibrous structures than radiofrequency ablation.
D) Cryoablation will not damage the skin immediately overlying a treated lesion.

Answer: B
Cryoablation freezes and destroys only water-containing cells (tumor and nerve
cells). A cryoablation lesion can be visualized in real time with CT because the ice ball
created around the cryoprobe is of lower density than water and surrounding nonfrozen
soft tissues. A radiofrequency ablation (RFA) lesion is not clearly visible on CT. The
leading edge of the ice ball causes shadowing on ultrasound [6, 11, 12]. Option B is the
best response.
Compared with RFA, cryoablation is less painful for the patient [13]. Option A is not
the best response. Collagenous or fibrous structures, including pleura, peritoneum, bowel
walls, bladder walls, and urothelium, are better preserved with cryoablation. Option C is
not the best response. Cryoablation can cause frostbite to nearby skin, whereas RFA can
burn overlying skin. Option D is not the best response.
60- Which arthropathy is LEAST likely to be associated with erosive
disease in the distal interphalangeal joints?

A) Psoriatic arthritis
B) Erosive osteoarthritis
C) Gout
D) Rheumatoid arthritis [86 %]

Answer: D
Rheumatoid arthritis usually does not cause erosive disease in the distal interphalangeal
joint. Exceptions to this rule are rare and but can be seen in patients with long standing
disease and a diagnosis on rheumatoid arthritis has been well established. Erosions can
also be seen in the distal interphalangeal joints of patients with rheumatoid arthritis that
also have erosive osteoarthritis (EOA). The erosions of EOA however will be central rather
than marginal and will be associated with osteophyte formation.

Reference:

Resnick, D. Rheumatoid arthritis. In: Diagnosis of Bone and Joint Disorders, 4rth ed.
Philadelphia,
PA: WB Saunders, 2002; 891-987.

61- Soft tissue swelling of an entire digit (sausage digit) is a relatively


specific finding for which arthropathy?

A) Psoriatic arthropathy [99 %]


B) Osteoarthritis
C) Gout
D) Rheumatoid arthritis

Answer: A
Soft tissue swelling of entire digit, also known as dactylitis, is a finding seen in
spondyloarthopathies such as psoriatic and reactive arthritis. It is not seen in rheumatoid
arthritis, gout or osteoarthritis. Dactylitis is seen in 16-24% of patients with psoriatic
arthritis and the reason for diffuse soft tissue swelling in these patients is unclear. Some
authors consider dactylitis to be a manifestation of enthesitis and other, a presentation of
flexor tenosynovitis. Regardless of the etiology, dactylitis is a relatively specific finding for
spondyloarthropathy such as psoriatic arthritis.

Reference:

Resnick, D. Rheumatoid arthritis. In: Diagnosis of Bone and Joint Disorders, 4rth
ed. Philadelphia, PA: WB Saunders, 2002; 1082-1109.
62- State-of-the-art ultrasound evaluation of patients with Rheumatoid
Arthritis requires:

A) Specialized ultrasound equipment


B) Dedicated image analysis software
C) Use of injectable sonographic contrast media
D) All of the Above
E) None of the above [68 %]

Answer: E
State-of-the-art ultrasound evaluation of patients can be performed with
conventional and commercially-available equipment. Postprocessing image
quantification, which is usually done with dedicated image analysis software, is not part
of the routine clinical examination, nor is the use of injectable sonographic contrast
media.

Reference:

Bruno, M.A., Gold, G.E., Mosher, T.J. “Arthritis In Color: Advanced Imaging In Arthritis,”
with 5 Contributors. Philadelphia: Saunders-Elsevier Scientific Press. c 2009. ISBN: 978-
14160-4722-3. Chapter 5: Ultrasound In Arthritis.

63- Advantages of Ultrasound over MRI for evaluation of patients with RA


include:

A) US has lower cost than MRI.


B) US has higher spatial resolution than MRI.
C) US is easier for patient to tolerate/cooperate with examination.
D) All of the above. [73 %]
E) A and C only.

Answer: D
Many radiologists do not realize that ultrasound actually has higher spatial resolution
than MRI, a point which is made in the lecture. Ultrasound examinations do not require
difficult positioning or require the patient to remain still for an extended period of time.
Their cost is usually much less than MRI as well.

Reference:

Bruno, M.A., Gold, G.E., Mosher, T.J. “Arthritis in Color: Advanced Imaging in Arthritis,”
with 5 Contributors. Philadelphia: Saunders-Elsevier Scientific Press. c 2009. ISBN: 978-
14160-4722-3 Chapter 5: Ultrasound in Arthritis.
64- Molecular / Nuclear imaging of patients with RA

A) Is entirely experimental at this time—with no role in the clinical management of


patients with RA
B) Primarily consists of “physiological” or “functional” imaging, rather than
“anatomic” or “structural” imaging methods [84 %]
C) Primarily involves use of highly specific, experimental “activatable” targeted
molecular probes
D) Currently limited to animal models and clinical trials

Answer: B
Although PET/CT, SPECT/CT and PET/MRI combine both anatomic and
physiologic/functional imaging, the main advantage of molecular techniques is to enable
the visualization of physiological, metabolic and cellular processes.

Reference:

Biswal, S. Chapter 9: “Molecular Imaging in Arthritis” in Bruno, M.A., Gold, G.E., Mosher,
T.J. “Arthritis in Color: Advanced Imaging in Arthritis,” with 5 Contributors.
Philadelphia: Saunders-Elsevier Scientific Press. c 2009. ISBN: 978-14160-4722-3.

65- All of the following can simulate MRI findings of Ankylosing


Spondylitis EXCEPT:

A) Ulcerative Colitis
B) Crohn’s Disease
C) Splenectomy [93 %]
D) Salmonella, Shigella, Yersinia infections
E) Intestinal bypass surgery

Answer: C
A multitude of disease processes can lead to enteropathic arthropathy, which has
similar imaging features as Ankylosing Spondylitis. These etiologies include ulcerative
colitis, Crohn’s disease, gastrointestinal infections (such as Salmonella, Shigella and
Yersinia) as well as intestinal bypass surgery. Splenectomy has not been reported to be
associated with enteropathic spondyloarthropathy.

Reference:

Mester AR, Mako EK, Karlinger K, et al. Enteropathic arthritis in the sacroiliac joint.
Imaging and differential diagnosis. Eur J Radiol 2000; 35:199-208.
66- Is synovial chondromatosis considered a metaplastic process of the
synovium?

A) True
B) False [62 %]

Answer: B
Although primary synovial chondromatosis historically has been deemed a
metaplastic process of the synovium, it is now widely considered an uncommon benign
neoplastic process in which hyaline cartilage nodules are present in the subsynovial
tissues of joints, tendon sheaths or bursae. The cartilaginous nodules are most often
found in the knee and hip, may enlarge and detach from the synovium. Rarely, there may
be malignant transformation of synovial chondromatosis into chondrosarcoma.

Reference:

Murphey MD, Vidal JA, Fanburg-Smith JC, Gajewski DA. Imaging of synovial
chondromatosis with radiologic and pathologic correlation. RadioGraphics 2007;
27:1465-88.

67- Which of the following is NOT associated with osteoarthrosis:

A) Osteophytes
B) Articular cartilage loss
C) Osteopenia [85 %]
D) Synovitis
E) Effusions

Answer: C
Osteoarthrosis is a degenerative process in which there is loss of articular cartilage,
resulting in increased stresses across a joint. The abnormal mechanical forces can lead to
reactive changes along the joint and increased (not decreased) osseous density related to
marginal osteophytes and subchondral sclerosis. There is an increased incidence of
synovitis and joint effusions in patients with osteoarthrosis, although these are
nonspecific findings and can be found in multiple other arthropathies.

Reference:

Brower AC and Flemming DJ. Osteoarthritis in Arthritis: In Black and White. 2nd ed.
Brower AC and Flemming DJ, ed. 1996. Elsevier Health Sciences.
68- Neonatal Medical and Surgical Chest Diseases A newborn chest shows
diffuse opacification with “ground-glass” appearance. Which one
question is most helpful in making a correct diagnosis?

A) What is the oxygen saturation?


B) Were there fetal decelerations?
C) Is there a fever?
D) What is the gestational age? [85 %]
E) Is there a murmur?

Answer: D

The ground-glass pattern is seen with Respiratory Distress Syndrome (RDS) or


Surfactant Deficient Disease (SDD) and Group B Streptococcal infection. RDS is most
common with prematurity. Therefore, the most helpful information is gestational age.
Option A is incorrect because: in both cases of RDS and Group B Streptococcal infection
there may be oxygen desaturation Option B is incorrect because: RDS and Group B
Streptococcal infection do not present in the fetal period Option C is incorrect because:
Fever is not a good indicator for infection in the neonatal period as the newborn may not
mount a response Option E is incorrect because: Murmur is not a feature of either RDS
or Group B Streptococcal infection

References:

1. Agrons GA et al. Lung Disease in Premature Neonates: Radiologic-Pathologic


Correlation. Radiographics 2005; 25:1047-1073.
2. Donnelly L (ed). Diagnostic Imaging: Pediatrics. Surfactant Deficient Disease. 2:30-33.
Amirsys, Salt Lake City, Utah 2005.
69- With ground glass, what one is the most helpful ancillary finding?

A) Pneumothorax
B) Pleural effusion [77 %]
C) Pneumomediastinum
D) Cardiomegaly
E) Lymphadenopathy

Answer: B

The two most common reasons for ground-glass pattern in the newborn are RDS and
Group B Streptococcal infection. Pleural effusion is unusual in RDS and
common in infection. Option A is incorrect because: Pneumothorax may be seen as a later
complication of RDS or Group B Streptococcal infection and is not a good discriminator
between the two. Option C is incorrect because: Pneumomediastinum may be seen as a
later complication of RDS or Group B Streptococcal infection and is not a good
discriminator between the two. Option D is incorrect because: Cardiomegaly is not a
common feature of RDS or Group B Streptococcal
infection. Option E is incorrect because: Lymphadenopathy is not a feature of RDS or
Group B Streptococcal infection and would be extremely difficult to see on newborn chest
radiography as would be obscured by thymus.

References:

1. Donnelly L (ed). Diagnostic Imaging: Pediatrics. Surfactant Deficient Disease and


Neonatal Pneumonia. 2:30-37. Amirsys, Salt Lake City, Utah 2005.
2. Cleveland RH. A radiologic update on medical diseases of the newborn chest. Pediatric
Radiology 1995; 25(8): 631-7
70- A newborn has aspirated thick meconium and has respiratory distress.
What pattern do you expect to see on CXR?

A) Ground-glass
B) Fine Reticular
C) Coarse Reticular [100 %]
D) Mass
E) Multiple Lucencies

Answer: C

Meconium aspiration results in chemical pneumonitis and inspissated secretions,


which will lead to areas of atelectasis and interstitial inflammation seen as coarse
interstitial disease. In fact, some of the opacity lies within the airspaces but the pattern is
most strikingly interstitial Option A is incorrect because: Ground glass pattern occurs in
RDS or Group B Streptococcal Infection Option B is incorrect because: Fine reticular
pattern is seen with edema, retained fetal lung liquid and some cases of infection.
Option D is incorrect because: Meconium aspiration does not produce masses.
Option E is incorrect because: Meconium aspiration may cause pneumothorax in up to
25% of cases but this rarely presents as multiple lucencies (suggesting interstitial
emphysema).

References:

1. Cleveland RH. A radiologic update on medical diseases of the newborn chest. Pediatric
Radiology 1995; 25(8): 631-7 Donnelly L (ed). Diagnostic Imaging: Pediatrics. Meconium
Aspiration. 2:38-41. Amirsys, Salt Lake City, Utah 2005.
71- All of the following are diagnosed prenatally by ultrasound (US) or
MRI except:

A) Pulmonary interstitial emphysema [85 %]


B) Pulmonary sequestration
C) Congenital diaphragmatic hernia
D) Congenital pulmonary airway malformation
E) Bronchial atresia

Answer: A

All but pulmonary interstitial emphysema develops in the prenatal period and can
be diagnosed by imaging. Pulmonary interstitial emphysema is part of the air block
sequence and is seen once the child starts breathing. Option B is incorrect because:
Pulmonary sequestration can be readily diagnosed and characterized by prenatal US and
MRI Option C is incorrect because: Congenital diaphragmatic hernia can be readily
diagnosed and characterized by prenatal US and MRI Option D is incorrect because:
Congenital cystic adenomatoid malformation can be readily diagnosed and characterized
by prenatal US and MRI Option E is incorrect because: Bronchial atresia can be suspected
and characterized by prenatal US and MRI

References:

1. Agrons GA et al. Lung Disease in Premature Neonates: Radiologic-Pathologic


Correlation. Radiographics 2005; 25:1047-1073.
2. Barnes NA, Pilling DW. Bronchopulmoary foregut malformations: embryology,
radiology and quandary. Eur Radiol. 2003 Dec;13(12):2659-73.
72- The pattern of apparent cardiomegaly with increased pulmonary
blood flow is seen with all but:

A) Ventricular Septal Defect (VSD)


B) Patent ductus arteriosus (PDA)
C) Hypoplastic left heart syndrome (HLHS)
D) Transient tachypnea of the newborn (TTN)
E) Respiratory distress syndrome (RDS) [62 %]

Answer: A

All but pulmonary interstitial emphysema develops in the prenatal period and can
be diagnosed by imaging. Pulmonary interstitial emphysema is part of the air block
sequence and is seen once the child starts breathing. Option B is incorrect because:
Pulmonary sequestration can be readily diagnosed and characterized by prenatal US and
MRI
Option C is incorrect because: Congenital diaphragmatic hernia can be readily
diagnosed and characterized by prenatal US and MRI Option D is incorrect because:
Congenital cystic adenomatoid malformation can be readily diagnosed and characterized
by prenatal US and MRI Option E is incorrect because Bronchial atresia can be suspected
and characterized by prenatal US and MRI

References:

1. Agrons GA et al. Lung Disease in Premature Neonates: Radiologic-Pathologic


Correlation. Radiographics 2005; 25:1047-1073.
2. Barnes NA, Pilling DW. Bronchopulmonary foregut malformations: embryology,
radiology and quandary. Eur Radiol. 2003 Dec;13(12):2659-73.
73- All of the following are possible complications for bacterial infection in
children except:

A) Necrotizing pneumonia
B) Pleuropulmonary blastoma [85 %]
C) Lung abscess
D) Pneumatocele

Answer: E

Cardiomegaly is uncommon in uncomplicated RDS. Once the child begins to


improve, a patent ductus arteriosus may declare itself as the pulmonary arterial pressures
decrease and then there may be a big heart and increased pulmonary blood flow. Option
A is incorrect because: VSD has cardiomegaly and increased pulmonary arterial blood
flow Option B is incorrect because: PDA has cardiomegaly and increased pulmonary
arterial blood flow Option C is incorrect because: HLHS has cardiomegaly and increased
pulmonary venous blood flow with edema Option D is incorrect because: TTN has
cardiomegaly and increased interstitial markings, which leads to apparent increased
venous blood flow

References:

1. Agrons GA et al. Lung Disease in Premature Neonates: Radiologic-Pathologic


Correlation. Radiographics 2005; 25:1047-1073.
2. Donnelly L (ed). Diagnostic Imaging: Pediatrics. Surfactant Deficient Disease. 2:30-33.
Amirsys, Salt Lake City, Utah 2005.
74- Recurrent infection in the left lower lobe in children is most likely due
to which one of the following diagnoses?

A) Viral bronchiolitis
B) Primary tuberculosis infection
C) Histoplasmosis infection
D) Pulmonary sequestration [92 %]

Answer: D

Underlying congenital lung anomalies such as pulmonary sequestration often


cause recurrent infection in children. When recurrent infection repeatedly occurs,
particularly within the lower lobes (left lower lobe > right lower lobe) which is a common
location for pulmonary sequestration, pulmonary sequestration should be considered as
the cause of recurrent infection in children.
Option A is incorrect because: viral bronchiolitis does not typically present as
recurrent infection in the lower lobe. Bilateral perihilar interstitial opacities associated
with peribronchial cuffing and hyperinflation in infants or young children are typical
radiological findings.
Option B is incorrect because: a primary tuberculosis infection typically manifests
as mediastinal lymphadenopathy with or without parenchymal lung disease, which is not
localized within the left lower lobe. Option C is incorrect because: a histoplasmosis
infection usually presents as bilateral lung nodules and mediastinal / hilar
lymphadenopathy which can be often calcified. Isolated histoplasmosis infection within
the left lower lobe is unlikely.

References:

1. Yikilmaz A, Lee EY. CT imaging of mass-like nonvascular pulmonary lesions in children.


Pediatr Radiol. 2007; 37(12): 1253 – 1263.
2. Lee EY, Boiselle PM, Cleveland RH. Multidetector CT evaluation of congenital lung
anomalies. Radiology. 2008; 247(3): 632 – 648.
75- All of the following are underlying risk factors for invasive pulmonary
aspergillosis infection in children except:

A) Neutropenia
B) Corticosteroid therapy
C) History of prior prematurity [92 %]
D) AIDS

Answer: C

Invasive pulmonary aspergillosis infection typically occurs in


immunocompromised or immunosuppressed children. History of prior prematurity by
itself will not cause immunocompromise or immunosuppression in children. Option A is
incorrect because: children with neutropenia, characterized by an abnormally low
number of neutrophils, a type of white blood cell, are prone to infectious processes
including fungal infection such as aspergillosis infection.
Option B is incorrect because: children receiving corticosteroid therapy are often
under the immunosuppressed state. Option D is incorrect because: children with AIDS
infection caused by human immunodeficiency virus (HIV) have decreased effectiveness
of the immune system, resulting in susceptibility to various infectious processes including
fungal infection.

Reference:

1. Yikilmaz A, Lee EY. CT imaging of mass-like nonvascular pulmonary lesions in children.


Pediatr Radiol. 2007; 37(12): 1253 – 1263.
76- What is the most definitive test for diagnosing CNS vasculitis?

A) CT angiography with 3D volume rendering.


B) Cerebral angiography.
C) Biopsy. [91 %]
D) MR angiography.
E) Transcranial Doppler sonography.

Answer: C

Because many cases of vasculitis affect small to medium sized blood vessels, MR
angiography is relatively insensitive for the diagnosis of CNS vasculitis. CT angiography
is less accurate than cerebral angiography.
Cerebral angiography is used as the gold standard for diagnosis at many
institutions. When characteristic angiographic findings such as alternating areas of
stenosis and dilatation or a beading appearance are observed in multiple vessels and
multiple vascular beds, cerebral angiography is diagnostic. However, angiography can be
normal in up to 40% of biopsy-proven cases. Thus, negative cerebral angiography does
not completely exclude the diagnosis. Moreover, its specificity is not perfect. Intracranial
atherosclerotic disease may show irregularity of multiple vessels, mimicking CNS
vasculitis.
Transcranial Doppler sonography is used to characterize the morphology of the
superficial temporal artery as a screening tool for temporal arteritis. However,
transcranial Doppler sonography is not suitable to evaluate arteries fully to diagnose CNS
vasculitis. Options A, B, D, and E are not the best responses.
Biopsy of CNS tissue would logically be considered the ultimate gold standard of
diagnosis, but clearly the procedure is limited by several factors. It is highly invasive and
carries certain risks. Successful biopsy requires a willing and experienced neurosurgeon,
who may not be readily available.
Sampling error could result in limited sensitivity. The site of biopsy should be
tailored to the individual patient [2]. The biopsy of the superficial temporal artery is often
performed in patients suspected of having giant cell vasculitis. The false-negative rate of
biopsy for a diagnosis of vasculitis has been reported to be 16%, yielding a sensitivity of
84% [2]. Therefore, the most definitive test is biopsy of the blood vessels. Option C is
the best response.
77- Which one of the following is NOT associated with vasculitis?

A) Polyarteritis nodosa.
B) Tuberculosis.
C) Systemic lupus erythematosus.
D) Drugs (amphetamine, cocaine).
E) Marfan syndrome. [97 %]

Answer: E

The causes of vasculitis are often divided into infectious and noninfectious.
Infectious causes include tuberculosis, fungal infection, bacterial vasculitis, and HIV
vasculitis. The frequent causes of noninfectious vasculitis are immune mediated vasculitis
such as rheumatoid arthritis and systemic lupus erythematosus, and granulomatous
disease, such as Wegener’s granulomatosis and sarcoid, as well as giant cell arteritis (also
called temporal arteritis). Drugs, particularly amphetamines and cocaine, are frequent
causes of noninfectious vasculitis. An amphetamine causes inflammatory vasculitis with
vascular wall necrosis and subsequent hemorrhage.
The pathologic features of amphetamine related vasculitis are similar to those of
polyarteritis nodosa. This patient had taken amphetamines for several years. Cocaine, on
the other hand, induces cerebral infarction or ischemia as well as hemorrhage by
vasoconstrictive effect and increased platelet aggregation, rather than a vasculitis-type
inflammation of the vessels [1]. Options A, B, C, and D are not the best responses.
Marfan syndrome is an autosomal dominant disorder of the connective tissue
characterized by disproportionally long limbs and tall stature. It affects the heart and
aorta and causes aortic root dilatation, aortic regurgitation, and dissection. However,
involvement of CNS vessels is unusual. Option E is the best
response.
78- Which one of the following techniques is LEAST appropriate to confirm
a diagnosis of venous sinus thrombosis?

A) Rapid contrast-enhanced gradient-echo MRI with contrast-enhanced multiplanar


imaging.
B) MDCT angiography or venography.
C) Brain MRI and phase-contrast MR venography.
D) Cerebral angiography. [96 %]
E) Brain MRI with time-of-flight MR venography.

Answer: D

The next diagnostic test to confirm venous sinus thrombosis is brain MRI with MR
venography. On conventional MRI, venous sinus thrombosis may be suspected from lack
of a flow void or high-signal thrombus in the dural sinuses. Lack of flow void is best
appreciated on FLAIR or T2- weighted spin-echo images. Parenchymal changes, such as
venous infarction or hemorrhage, along with lack of a flow void on conventional MRI
raises the suspicion of venous sinus thrombosis. Superacute thrombus is relatively
isointense on T1- and hypointense on T2-weighted images because of deoxyhemoglobin
potentially mimicking slow venous flow on conventional MR images and thus requiring
MR venography to confirm the diagnosis of venous sinus thrombosis [3].
A subacute blood clot in the venous sinus is often seen as hyperintense on T1-
weighted images as a result of methemoglobin mimicking flowing blood on time-off light
(TOF) MR venography. Phase-contrast MR venography is a fairly reliable test for the
diagnosis of subacute venous sinus thrombosis.
Recently, rapid contrast-enhanced gradient-echo imaging has been reported to yield
a higher diagnostic accuracy than 2D TOF MR venography [4, 5]. With advances in MDCT
technology, CT angiography and venography have been increasingly used to diagnose
venous sinus thrombosis and are considered a quick and reliable alternative to MR
venography. Options A, B, C, and E, the appropriate next tests to confirm venous sinus
thrombosis, are not the best responses.
Cerebral angiography also shows lack of flow in the dural venous sinuses. However,
cerebral angiography is an invasive test that is associated with complication rates of 1–
2%. Noninvasive imaging should be considered first to confirm the diagnosis of venous
sinus thrombosis. Option D, which is the least appropriate technique, is the best
response.
79- Which one of the following is NOT associated with venous sinus
thrombosis?

A) Sickle cell disease.


B) Oral contraceptives.
C) Cancer.
D) Disseminated intravascular coagulation. [97 %]
E) Dehydration.

Answer: D

Common medical conditions associated with venous sinus thrombosis are


pregnancy, a postpartum state, and hypercoagulable states such as sickle cell disease, oral
contraceptives use, and cancer. Dehydration often seen in neonates and elderly patients
is associated with venous sinus thrombosis as a result of low flow circulatory states.
Extrinsic compression (tumor) or adjacent infection (mastoiditis) is also a risk factor.
Options A, B, C, and E are not the best responses.
Disseminated intravascular coagulation, however, is a hypocoagulable state and is
not associated with venous sinus thrombosis. Option D is the best response.
Venous sinus thrombosis is an underdiagnosed condition and can be a life-
threatening disease if left untreated. Delay in diagnosis often leads to rapid deterioration
and poor clinical outcomes. Intracranial hemorrhage is associated with 20–50% of cases
of venous sinus thrombosis. Treatment for venous sinus thrombosis is immediate IV
anticoagulation with low-molecular-weight heparin, despite the risk of hemorrhagic
complications [6].

80- What is the 30-day mortality rate of subarachnoid hemorrhage


secondary to aneurysm?

A) 15%.
B) 30%.
C) 45%. [95 %]
D) 65%.
E) 80%.

Answer: C

Aneurysmal subarachnoid hemorrhage (SAH) has a 30-day mortality rate of 45%,


with approximately half of the survivors sustaining irreversible brain damage [7]. Option
C is the best response.
The annual incidence of aneurysmal SAH is six per 100,000 in the United States.
Approximately 5–10% of stroke cases are secondary to ruptured saccular aneurysms [8].
Recurrent hemorrhage remains a serious consequence, with a 70% fatality rate in patients
who rebleed.
81- Which of the following factors does NOT influence management
decisions for a ruptured aneurysm?

A) Age.
B) Sex. [99 %]
C) Aneurysm size.
D) Aneurysm location.
E) History of hypertension.

Answer: B

Treatment options for a ruptured aneurysm are surgical resection or endovascular


coil embolization. Endovascular occlusion of aneurysms using electrolytically detachable
Guglielmi detachable coil system (GDC, Target Therapeutics [now Boston Scientific]) has
been used to treat ruptured or unruptured aneurysms in a large number of patients
worldwide. Published reports suggest that the endovascular technique is associated with
fewer treatment-related complications than open surgery [7]. However, the long-term
efficacy in the prevention of rupture or recurrence of aneurysm remains indeterminate.
The recent International Subarachnoid Aneurysm Trial (ISAT) showed that retreatment
was performed in 17.4% of patients treated with endovascular coiling and in 3.8% of
patients after surgical clipping [9].
Younger age and larger aneurysm size were risk factors for retreatment. A higher
rate of recurrence is seen in posterior communicating artery aneurysms after
endovascular coiling and anterior communicating artery aneurysms after surgical
clipping, which might reflect the technical difficulty. Hypertension is also associated with
an increasing rate of rupture in patients with an unruptured aneurysm. Thus, age, medical
history such as hypertension, aneurysm location, and size are all relevant factors in
treatment decisions and, therefore, options A, C, D, and E, all relevant factors in
treatment decisions, are not the best responses.
Although aneurysms are more common in women than in men, sex is not a factor
affecting management decisions or predicting outcomes. Option B is the best
response. Despite CT angiography showing an aneurysm causing subarachnoid
hemorrhage, cerebral angiography is still performed at some institutions.
82- Which one of the following is NOT a rationale for performing cerebral
angiography in this setting?

A) Searching for an additional incidental aneurysm that could be treated at the same
time.
B) Better assessing the degree of incorporation of the aneurysm wall into the parent
vessel.
C) Assessing flow dynamics—that is, the side of the internal carotid artery feeding
anterior communicating aneurysm.
D) Measuring the aneurysm neck–dome ratio. [94 %]
E) Suspecting mycotic aneurysm in patients with IV drug use.

Answer: D

CT angiography is a noninvasive vascular imaging technique that has replaced


catheter angiography in some institutions. CT angiography may show aneurysms larger
than 3 mm with a sensitivity of 77–97% and specificity of 87–100% [10]. CT angiography
also has been used as a screening tool in populations at high risk for cerebral aneurysms.
Cerebral angiography, however, still remains the gold standard in the diagnostic
evaluation of cerebral aneurysms. In particular, 3D rotational angiography shows the
most information about small perforating vessels, the relationship of the aneurysm to the
parent vessels (how much of the aneurysm wall is incorporated into the parent vessel),
and the flow dynamics of aneurysms that affect surgical planning.
Multiple aneurysms can be seen in patients with subarachnoid hemorrhage.
These incidental aneurysms are often smaller than 3 mm, which CT angiography has a
limited ability to detect. Small unruptured aneurysms are associated with a risk of future
SAHs. If they are in a surgically or endovascularly accessible location, these incidental
aneurysms can be treated at the same time as the ruptured primary aneurysm.
Mycotic aneurysms often involve peripheral vessels and may present with
subarachnoid or parenchymal hemorrhage or septic emboli. Because mycotic aneurysms
involve distal vessels, cerebral angiography is a more definitive test than CT angiography.
Options A, B, C, and E are not the best responses. The aneurysm neck–dome ratio can be
calculated on the basis of CT angiography. Option D is the best response.

83- What percentage of patients with acute ischemic stroke are treated
with IV tPA?

A) Less than 6%. [96 %]


B) 8–15%.
C) 20–25%.
D) 25–50%.
E) 50–60%.

Answer: A
84- Which of the following is NOT associated with poor neurologic
outcomes in patients with acute ischemic infarction?

A) Hypoventilation.
B) Extensive area of low attenuation and mass effect on initial head CT.
C) Hyperglycemia.
D) Hypothermia. [97 %]
E) Arrhythmia.

Answer: D

Maintaining adequate tissue oxygenation is critical in the setting of acute stroke to


prevent hypoxia and potential worsening of brain damage. Patients with decreased
consciousness or brain stem dysfunction have the greatest risk of airway compromise.
The prognosis of a patient who requires endotracheal intubation is generally poor;
approximately 50% of these patients die within 30 days of their stroke. Option A is not
best response.
An extensive area of low attenuation on initial head CT indicates widespread
damage to the brain tissue. An “early infarct sign” on unenhanced CT involving more than
one third of the territory of the middle cerebral infarction indicates a poor outcome. The
presence of mass effect or edema is also associated with an eightfold increase in the risk
of symptomatic hemorrhage [11]. Option B is not the best response.
Hyperglycemia is associated with poor clinical outcomes, presumably due to
increased tissue acidosis secondary to anaerobic glycolysis and lactic acidosis.
Hyperglycemia may affect the blood–brain barrier and lead to brain edema.
Hypoglycemia may cause focal neurologic signs and symptoms that mimic acute ischemic
stroke. Hypoglycemia itself may aggravate neuronal ischemia. The prompt assessment of
the serum glucose level and correction of the glucose level are important. Option C is not
the best response.
Fever in the setting of acute ischemic stroke is associated with a poor neurologic
outcome secondary to increased metabolic demands and enhanced release of
neurotransmitters. Hypothermia is not associated with poor clinical outcomes. In fact,
hypothermia has been reported to be neuroprotective in experimental models and small
clinical trials. Hypothermia may delay depletion of the energy reserve, slow tissue
acidosis, and slow calcium iron influx into cells. Option D is the best response.
Patients with acute ischemic stroke have an increased risk of developing
myocardial infarction and cardiac arrhythmia. Patients with infarctions of the right
hemisphere, particularly those involving the insula, may have an increased risk of cardiac
complications, presumably secondary to disturbances in autonomic nervous system
function. The most common arrhythmia associated with acute stroke is atrial fibrillation,
which may be either the cause of stroke or a complication. Life-threatening arrhythmia is
relatively uncommon, but sudden death may occur. Cardiac monitoring is often required
for at least first 24 hours after the onset of stroke symptoms. Option E is not the best
response.
85- Which of the following in a patient’s medical history is LEAST likely to
be associated with septic emboli in the brain?

A) Organ transplantation with pulmonary infection.


B) Cancer and presently receiving systemic chemotherapy.
C) IV drug abuse.
D) Recent travel to Southeast Asia. [97 %]
E) Aortic valve replacement and endocarditis.

Answer: D

Septic emboli are often seen in immunocompromised patients such as those who
have undergone organ transplantation, those who have AIDS, and patients who have
undergone chemotherapy. In these cases, organisms may include tuberculosis or fungal
infections. Among immunocompetent patients, infection with Staphylococcus organisms
is most often seen in IV drug abusers or in patients with endocarditis. Options A, B, C,
and E are not the best responses.
A history of recent travel to Southeast Asia can be seen in other infections such as
tuberculosis, brucellosis, West Nile virus, hepatitis, and malaria. This is not the expected
history in this patient. Option D is the best response.
This patient had a history of heart transplantation and pulmonary aspergillosis.
Pulmonary aspergillosis in severely immunocompromised patients is highly invasive and
has a dismal prognosis (near 100% mortality). It quickly gains access to the systemic
circulation and is disseminated throughout the body, including the brain. This patient
died 4 days after MRI was performed.
86- What are the characteristic MRI findings of disseminated cerebral
aspergillosis in immunocompromised patients?

A) Numerous foci of restricted diffusion in the corticomedullary junction, basal


ganglia, and thalami, with minimum or no enhancement. [97 %]
B) Markedly bright signal on diffusion-weighted images.
C) Involvement of the middle cerebellar peduncle.
D) Leptomeningeal invasion.
E) Infection in the paranasal sinuses.

Answer: A

Disseminated cerebral aspergillosis infection in immunocompromised patients is most


often caused by hematogenous spread from pulmonary infection. Hematogenous, or
angioinvasive, Aspergillus organisms characteristically lodge inside medium-sized blood
vessels, resulting in multifocal infarction, and then invade through the vascular walls,
causing hemorrhagic transformation or direct extension into the parenchyma. This
vasculopathy-mediated septic infarction has regional vulnerability to basal ganglia or
thalami, in addition to the corticomedullary junction. The predilection to basal ganglia
and thalami indicates involvement of the lenticulostriate and thalamoperforating
arteries. Aspergillosis often destroys the internal elastic lamina of the cerebral arteries.
Perforating vessels are the first ones to lose their patency because of their narrow
diameter. MRI characteristics of disseminated aspergillosis involvement of the brain in
18 patients was reported by De- Lone et al. [16] and others [17, 18]. Those authors
reported that the typical MRI appearance is a predilection to basal ganglia or thalami.
Enhancement was minimal or absent. Lack of enhancement is most likely related to the
host’s immune capacity. Severely immunocompromised patients have no or little immune
capacity to react to an infectious organism to form capsule or inflammatory response;
thus, lack of enhancement may indicate poor prognosis and rapid dissemination of
angioinvasive aspergillosis. Option A is the best response.
Marked bright signal on diffusion-weighted images in this patient likely reflects
infarction and cytotoxic edema. Numerous foci of restricted diffusion can be seen in
patients with embolic infarction, brain abscesses, and metastases from highly cellular
tumors [19]. Option B is not the best response. Neither involvement of the middle
cerebellar peduncle nor leptomeningeal involvement is a typical finding for disseminated
aspergillosis. Options C and D are not the best responses. Involvement of the paranasal
sinuses is often seen in diabetic patients who have angioinvasive mucormycosis.
Mucormycosis is a rare opportunistic infection caused by ubiquitous fungi typically found
in soil or dust. The route of infection is usually rhinocerebral and is commonly seen in
patients with uncontrolled diabetes, which is often associated with metabolic acidosis or
ketoacidosis [20]. Mucormycosis can spread from the paranasal sinuses to the brain in a
few days. Treatment should include aggressive debridement and IV amphotericin B.
Option E is not the best response.
87- Which of the followings is LEAST likely to be a risk factor for
Wernicke’s encephalopathy?

A) Chronic alcoholism.
B) Prolonged parenteral nutrition without a vitamin supplement.
C) Hyperemesis gravidarum.
D) Gastrectomy. [96 %]
E) Anorexia nervosa.

Answer: D

The triad of encephalopathy, ataxic gait, and oculomotor dysfunction is seen in


only one third of patients with Wernicke’s encephalopathy. Ocular abnormalities are the
hallmarks of Wernicke’s encephalopathy. The oculomotor signs are nystagmus, bilateral
lateral rectus palsies, and conjugate gaze palsies reflecting involvement of the oculomotor
and abducens nerves. Option D is the best response. Gait ataxia is believed to be due
to focal midline degeneration of the superior vermis, as opposed to global ataxia, a sign
of cerebellar dysfunction. Option E is not the best response.

Cerebellar testing with the finger-to-nose or heel-to-shin test may not elicit any
notable deficit. Vestibular dysfunction without hearing loss is also a common finding.
Rigidity and tremor as well as bradykinesia and postural instability are common
symptoms seen in patients with Parkinson’s disease, not in patients with Wernicke’s
encephalopathy. Visual hallucination is associated with psychiatric disorders and drugs,
particularly alcohol. Patients with schizophrenia often have visual and, more often,
auditory hallucinations.

Learning disability refers to a group of disorders affecting academic and


functional skills, including the abilities to listen, speak, write, read, and organize
information. It is not specific for Wernicke’s encephalopathy. Thus, options A, B, and C
are not the correct responses.

Вам также может понравиться